MS1Q

Pataasin ang iyong marka sa homework at exams ngayon gamit ang Quizwiz!

A nurse is developing a care plan for a client with hepatic encephalopathy. Which of the following are goals for the care for this client? Select all that apply. 1. Preventing constipation. 2. Administering lactulose (Cephulac). 3. Monitoring coordination while walking. 4. Checking the pupil reaction. 5. Providing food and fluids high in carbohydrate. 6. Encouraging physical activity

. 1, 2, 3, 4, 5. Constipation leads to increased ammonia production. Lactulose is a hyperosmotic laxative that reduces blood ammonia by acidifying the colon contents, which retards diffusion of nonionic ammonia from the colon to the blood while promoting its migration from the blood to the colon. Hepatic encephalopathy is considered a toxic or metabolic condition that causes cerebral edema; it affects a person's coordination and pupil reaction to light and accommodation. Food and fluids high in carbohydrates should be given because the liver is not synthesizing and storing glucose. Because exercise produces ammonia as a byproduct of metabolism, physical activity should be limited, not encouraged

The client is diagnosed with hypothyroidism. Which signs/symptoms should the nurse expect the client to exhibit? 1. Complaints of extreme fatigue and hair loss. 2. Exophthalmos and complaints of nervousness. 3. Complaints of profuse sweating and flushed skin. 4. Tetany and complaints of stiffness of the hands

1. A decrease in thyroid hormone causes decreased metabolism, which leads to fatigue and hair loss

The nurse is planning the care of a client diagnosed with Addison's disease. Which intervention should be included? 1. Administer steroid medications. 2. Place the client on fluid restriction. 3. Provide frequent stimulation. 4. Consult physical therapy for gait training

1. Clients diagnosed with Addison's disease have adrenal gland hypofunction. The hormones normally produced by the gland must be replaced. Steroids and androgens are produced by the adrenal gland.

Which of the following medications would the nurse question for a client with acute pancreatitis? 1. Furosemide (Lasix) 20 mg I.V. push. 2. Imipenem (Primaxin) 500 mg I.V. 3. Morphine Sulfate 2 mg I.V. push. 4. Famotidine (Pepcid) 20 mg I.V. push.

1. Furosemide can cause pancreatitis. Additionally, hypovolemia can develop with acute pancreatitis and Lasix will further delete fluid volume.

. The nurse is developing a plan of care for the client with viral hepatitis. The nurse should instruct the client to: 1. Obtain adequate bed rest. 2. Increase fluid intake. 3. Take antibiotic therapy as ordered. 4. Drink 8 oz of an electrolyte solution every day

1. Treatment of hepatitis consists primarily of bed rest with bathroom privileges. Bed rest is maintained during the acute phase to reduce metabolic demands on the liver, thus increasing its blood supply and promoting liver cell regeneration. When activity is gradually resumed, the client should be taught to rest before becoming overly tired. Although adequate fl uid intake is important, it is not necessary to force fl uids to treat hepatitis. Antibiotics are not used to treat hepatitis. Electrolyte imbalances are not typical of hepatitis.

A client with jaundice has pruritis and states that he has areas of irritation from scratching. What measures can the nurse discuss to prevent skin breakdown? Select all that apply. 1. Avoid lotions containing calamine. 2. Take baking soda baths. 3. Keep nails short and clean. 4. Rub with knuckles instead of nails. 5. Massage skin with alcohol. 6. Increase sodium intake in diet.

2, 3, 4. Baking soda baths can decrease pruritis. Keeping nails short and rubbing with knuckles can decrease breakdown when scratching cannot be resisted, such as during sleep. Calamine lotions help relieve itching. Alcohol will increase skin dryness. Sodium in the diet will increase edema and weaken skin integrity.

A 34-year-old female is diagnosed with hypothyroidism. The nurse should assess the client for which of the following? Select all that apply. 1. Rapid pulse. 2. Decreased energy and fatigue. 3. Weight gain of 10 lb. 4. Fine, thin hair with hair loss. 5. Constipation. 6. Menorrhagia.

2, 3, 5, 6. Clients with hypothyroidism exhibit symptoms indicating a lack of thyroid hormone. Bradycardia, decreased energy and lethargy, memory problems, weight gain, coarse hair, constipation, and menorrhagia are common signs and symptoms of hypothyroidism.

The nurse teaches a client with diabetes mellitus about differentiating between hypoglycemia and ketoacidosis. The client demonstrates an understanding of the teaching by stating that a form of glucose should be taken if which symptom or symptoms develop? Select all that apply. 1. Polyuria 2. Shakiness 3. Palpitations 4. Blurred vision 5. Lightheadedness 6. Fruity breath odor

2, 3, 5. Shakiness, palpitations, and lightheadedness are signs/symptoms of hypoglycemia and would indicate the need for food or glucose. Polyuria, blurred vision, and a fruity breath odor are manifestations of hyperglycemia

The nurse is caring for a 59-year-old client who is admitted to the hospital with a possible hip fracture. Following the admission assessment, nurse determines that the client is obese. About which disorders should the nurse teach the client that are often associated with obesity? Select all that apply. 1. Stroke 2. Degenerative joint disease 3. Urinary retention 4. Mobility problems 5. Chronic cough

2, 4. A client who is overweight and/or obese is at an increased risk for degenerative joint disease and functional and mobility problems. There is no relationship between obesity and chronic cough or urinary retention. There may be some relationship indirectly to stroke

A client with Graves disease requests that the nurse discuss the laboratory tests of the client. Which statements would the nurse include? Select all that apply. 1. "Your TSH level is increased." 2. "Your thyroid antibodies test is increased." 3. "Your serum T4 is decreased." 4. "Your serum T3 is increased." 5. "Your T3 uptake is decreased."

2, 4. Graves disease, or primary hyperthyroidism, has alterations in normal lab work. With this condition, TSH levels are decreased. Thyroid antibodies, serum T4, serum T3, and T3uptake tests are all increased

The client has developed iatrogenic Cushing's disease. Which statement is the scientific rationale for the development of this diagnosis? 1. The client has an autoimmune problem causing the destruction of the adrenal cortex. 2. The client has been taking steroid medications for an extended period for another disease process. 3. The client has a pituitary gland tumor causing the adrenal glands to produce too much cortisol. 4. The client has developed an adrenal gland problem for which the health-care provider does not have an explanation.

2. "Iatrogenic" means a problem has been caused by a medical treatment or procedure—in this case, treatment with steroids for another problem. Clients taking steroids over a period of time develop the clinical manifestations of Cushing's disease. Disease processes for which long-term steroids are prescribed include chronic obstructive pulmonary

The nurse is planning care for an 86-year-old client with type 2 diabetes mellitus. Which nursing diagnosis would be most appropriate for this client? 1. Risk for Falls 2. Risk for Infection 3. Ineffective Tissue Perfusion: Cardiac 4. Impaired Tissue Integrity

2. A client with diabetes mellitus is at risk for infection. No other information is given in the question with regard to risk for falls, ineffective tissue perfusion, or impaired tissue integrity as potential nursing diagnoses.

The nurse should institute which of the following measures to prevent transmission of the hepatitis C virus to health care personnel? 1. Administering hepatitis C vaccine to all health care personnel. 2. Decreasing contact with blood and blood contaminated fluids. 3. Wearing gloves when emptying the bedpan. 4. Wearing a gown and mask when providing direct care.

2. Hepatitis C is usually transmitted through blood exposure or needlesticks. A hepatitis C vaccine is currently under development, but it is available for use. The first line of defense against hepatitis B is the hepatitis B vaccine. Hepatitis C is not transmitted through feces or urine. Wearing a gown and mask will not prevent transmission of the hepatitis C virus if the caregiver comes in contact with infected blood or needles

Which electrolyte replacement should the nurse anticipate being ordered by the health-care provider in the client diagnosed with diabetic ketoacidosis (DKA) who has just been admitted to the ICU? 1. Glucose. 2. Potassium. 3. Calcium. 4. Sodium

2. The client in DKA loses potassium from increased urinary output, acidosis, catabolic state, and vomiting. Replacement is essential for preventing cardiac dysrhythmias secondary to hypokalemia

The UAP on the medical floor tells the nurse the client diagnosed with DKA wants something else to eat for lunch. Which intervention should the nurse implement? 1. Instruct the UAP to get the client additional food. 2. Notify the dietitian about the client's request. 3. Request the HCP increase the client's caloric intake. 4. Tell the UAP the client cannot have anything else.

2. The client will not be compliant with the diet if he or she is still hungry. Therefore, the nurse should request the dietitian talk to the client to try to adjust the meals so the client will adhere to the diet.

When conducting a health history with a female client with thyrotoxicosis, the nurse should ask about which of the following changes in the menstrual cycle? 1. Dysmenorrhea. 2. Metrorrhagia. 3. Oligomenorrhea. 4. Menorrhagia.

3. A change in the menstrual interval, diminished menstrual flow (oligomenorrhea), or even the absence of menstruation (amenorrhea) may result from the hormonal imbalances of thyrotoxicosis. Oligomenorrhea in women and decreased libido and impotence in men are common features of thyrotoxicosis. Dysmenorrhea is painful menstruation. Metrorrhagia, blood loss between menstrual periods, is a symptom of hypothyroidism. Menorrhagia, excessive bleeding during menstrual periods, is a symptom of hypothyroidism

he nurse is admitting a client diagnosed with primary adrenal cortex insufficiency (Addison's disease). Which clinical manifestations should the nurse expect to assess? 1. Moon face, buffalo hump, and hyperglycemia. 2. Hirsutism, fever, and irritability. 3. Bronze pigmentation, hypotension, and anorexia. 4. Tachycardia, bulging eyes, and goiter.

3. Bronze pigmentation of the skin, particularly of the knuckles and other areas of skin creases, occurs in Addison's disease. Hypotension and anorexia also occur with Addison's disease.

A nurse is assessing a client with Addison's disease. The nurse should review laboratory reports for which condition? 1. Hypokalemia. 2. Hypernatremia. 3. Hypoglycemia. 4. Decreased blood urea nitrogen (BUN) level.

3. Decreased hepatic gluconeogenesis and increased tissue glucose uptake cause hypoglycemia in clients with Addison's disease. Hyperkalemia and hyponatremia are characteristic of Addison's disease. There is decreased renal perfusion and excretion of waste products

A client with ascites and peripheral edema is at risk for impaired skin integrity. To prevent skin breakdown, the nurse should: 1. Institute range-of-motion (ROM) exercise every 4 hours. 2. Massage the abdomen once a shift. 3. Use an alternating air pressure mattress. 4. Elevate the lower extremities.

3. Edematous tissue is easily traumatized and must receive meticulous care. An alternating air pressure mattress will help decrease pressure on the edematous tissue. ROM exercises are important to maintain joint function, but they do not necessarily prevent skin breakdown. When abdominal skin is stretched taut due to ascites, it must be cleaned very carefully. The abdomen should not be massaged. Elevation of the lower extremities promotes venous return and decreases swelling.

The nurse is assessing a client with cirrhosis who has developed hepatic encephalopathy. The nurse should notify the physician of a decrease in which lab serum that is a potential precipitating factor for hepatic encephalopathy? 1. Aldosterone. 2. Creatinine. 3. Potassium. 4. Protein.

3. Hypokalemia is a precipitating factor in hepatic encephalopathy. A decrease in creatinine results from muscle atrophy; an increase in creatinine would indicate renal insufficiency. With liver dysfunction increased aldosterone levels are seen. A decrease in serum protein will decrease colloid osmotic pressure and promote edema

Interferon alfa-2b (Intron A) has been prescribed to treat a client with chronic hepatitis B. The nurse should assess the client for which of the following adverse effects? 1. Retinopathy. 2. Constipation. 3. Flulike symptoms. 4. Hypoglycemia

3. Interferon alfa-2b most commonly causes flulike adverse effects, such as myalgia, arthralgia, headache, nausea, fever, and fatigue. Retinopathy is a potential adverse effect, but not a common one. Diarrhea may develop as an adverse effect. Clients are advised to administer the drug at bedtime and get adequate rest. Medications may be prescribed to treat the symptoms. The drug may also cause hematologic changes; therefore, laboratory tests such as a CBC and differential should be conducted monthly during drug therapy. Blood glucose lab values should be monitored for the development of hyperglycemia

The nurse is discussing postoperative care with a client scheduled for Roux-en-Y gastric bypass surgery. Which client statement indicates that learning goals for this client have been met? 1. "Maintaining protein intake will be a priority in my recovery diet." 2. "I will need to take daily vitamin and mineral supplements." 3. "I will initially take in only liquids, such as low-sugar juices." 4. "The foods I am allowed to eat gradually will be increased."

3. Liquids are initially the only intake allowed after bariatric surgery. Sugary beverages and fruit juices with added sugar should be avoided, as these products can promote dumping syndrome. Vitamin and mineral supplements will be prescribed after the immediate postoperative period

The client is admitted to the medical department with a diagnosis of rule-out (R/O) acute pancreatitis. Which laboratory values should the nurse monitor to confirm this diagnosis? 1. Creatinine and (BUN). 2. Troponin and (CK-MB). 3. Serum amylase and lipase. 4. Serum bilirubin and calcium.

3. Serum amylase increases within two (2) to 12 hours of the onset of acute pancreatitis to two (2) to three (3) times normal and returns to normal in three (3) to four (4) days; lipase elevates and remains elevated for seven (7) to 14 days.

he nurse is completing discharge teaching to the client diagnosed with acute pancreatitis. Which instruction should the nurse discuss with the client? 1. Instruct the client to decrease alcohol intake. 2. Explain the need to avoid all stress. 3. Discuss the importance of stopping smoking. 4. Teach the correct way to take pancreatic enzymes.

3. Smoking stimulates the pancreas to release pancreatic enzymes and should be stopped.

The client is admitted to the medical unit with a diagnosis of rule-out diabetes insipidus (DI). Which instructions should the nurse teach regarding a fluid deprivation test? 1. The client will be asked to drink 100 mL of fluid as rapidly as possible and then will not be allowed fluid for 24 hours. 2. The client will be administered an injection of antidiuretic hormone (ADH), and urine output will be measured for four (4) to six (6) hours. 3. The client will have nothing by mouth (NPO), and vital signs and weights will be done hourly until the end of the test. 4. An IV will be started with normal saline, and the client will be asked to try to hold the urine in the bladder until a sonogram can be done.

3. The client is deprived of all fluids, and if the client has DI the urine production will not diminish. Vital signs and weights are taken every hour to determine circulatory status. If a marked decrease in weight or vital signs occurs, the test is immediately terminated.

. The client diagnosed with hypothyroidism is prescribed the thyroid hormone levothyroxine (Synthroid). Which assessment data indicate the medication has been effective? 1. The client has a three (3)-pound weight gain. 2. The client has a decreased pulse rate. 3. The client's temperature is WNL. 4. The client denies any diaphoresis

3. The client with hypothyroidism frequently has a subnormal temperature, so a temperature WNL indicates the medication is effective.

The nurse develops a teaching plan for the client about how to prevent the transmission of hepatitis A. Which of the following discharge instructions is appropriate for the client? 1. Spray the house to eliminate infected insects. 2. Tell family members to try to stay away from the client. 3. Tell family members to wash their hands frequently. 4. Disinfect all clothing and eating utensils

3. The hepatitis A virus is transmitted via the fecal-oral route. It spreads through contaminated hands, water, and food, especially shellfish growing in contaminated water. Certain animal handlers are at risk for hepatitis A, particularly those handling primates. Frequent hand washing is probably the single most important preventive action. Insects do not transmit hepatitis A. Family members do not need to stay away from the client with hepatitis. It is not necessary to disinfect food and clothing.

The emergency department nurse is caring for a client diagnosed with HHNS who has a blood glucose of 680 mg/dL. Which question should the nurse ask the client to determine the cause of this acute complication? 1. "When is the last time you took your insulin?" 2. "When did you have your last meal?" 3. "Have you had some type of infection lately?" 4. "How long have you had diabetes?"

3. The most common precipitating factor is infection. The manifestations may be slow to appear, with onset ranging from 24 hours to two (2) weeks

The client diagnosed with type 1 diabetes has a glycosylated hemoglobin (A1c) of 8.1%. Which interpretation should the nurse make based on this result? 1. This result is below normal levels. 2. This result is within acceptable levels. 3. This result is above recommended levels. 4. This result is dangerously high

3. This result parallels a serum blood glucose level of approximately 180 to 200 mg/dL. An A1c is a blood test reflecting average blood glucose levels over a period of three (3) months; clients with elevated blood glucose levels are at risk for developing long-term complications.

A client who is recovering from hepatitis A has fatigue and malaise. The client asks the nurse, "When will my strength return?" Which of the following responses by the nurse is most appropriate? 1. "Your fatigue should be gone by now. We will evaluate you for a secondary infection." 2. "Your fatigue is an adverse effect of your drug therapy. It will disappear when your treatment regimen is complete." 3. "It is important for you to increase your activity level. That will help decrease your fatigue." 4. "It is normal for you to feel fatigued. The fatigue should go away in the next 2 to 4 months."

4. During the convalescent or post icteric stage of hepatitis, fatigue and malaise are the most common complaints. These symptoms usually disappear within 2 to 4 months. Fatigue and malaise are not evidence of a secondary infection. Hepatitis A is not treated by drug therapy. It is important that the client continue to balance activity with periods of rest.

The family of a client with cirrhosis of the liver asks what symptoms they need to look for while the client is being cared for in their home. What should the nurse teach the family that indicates portal hypertension in this client? 1. Muscle wasting 2. Hypothermia 3. Bleeding gums 4. Hemorrhoids

4. Obstruction to portal blood flow causes a rise in portal venous pressure, resulting in splenomegaly, ascites, and dilation of collateral venous channels predominately in the paraumbilical and hemorrhoidal veins and the cardia of the stomach, and extending into the esophagus. Bleeding gums indicate insufficient vitamin K production in the liver. Muscle wasting is commonly associated with the poor nutritional intake seen in clients with cirrhosis. Hypothermia is an unrelated finding

A client has undergone a laparoscopic cholecystectomy. Which of the following instructions should the nurse include in the discharge teaching? 1. Empty the bile bag daily. 2. If you become nauseated, breathe deeply into a paper bag. 3. Keep adhesive dressings in place for 6 weeks. 4. Report bile-colored drainage from any incision.

4. There should be no bile-colored drainage coming from any of the incisions postoperatively. A laparoscopic cholecystectomy does not involve a bile bag. Breathing deeply into a paper bag will prevent a person from passing out due to hyperventilation; it does not alleviate nausea. If the adhesive dressings have not already fallen off, they are removed by the surgeon in 7 to 10 days, not 6 weeks

Which arterial blood gas results should the nurse expect in the client diagnosed with diabetic ketoacidosis? 1. pH 7.34, Pao2 99, Paco2 48, HCO3 24. 2. pH 7.38, Pao2 95, Paco2 40, HCO3 22. 3. pH 7.46, Pao2 85, Paco2 30, HCO3 26. 4. pH 7.30, Pao2 90, Paco2 30, HCO3 18.

4. This ABG indicates metabolic acidosis, which is expected in a client diagnosed with diabetic ketoacidosis.

Which of the following interventions should the nurse anticipate incorporating into the client's plan of care when hepatic encephalopathy initially develops? 1. Inserting a nasogastric (NG) tube. 2. Restricting fluids to 1,000 mL/day. 3. Administering I.V. salt-poor albumin. 4. Implementing a low-protein diet.

4. When hepatic encephalopathy develops, measures are taken to reduce ammonia formation. Protein is restricted in the diet. An NG tube is not inserted initially but may be necessary as the disease progresses. Fluid restriction and salt-poor albumin are incorporated into the treatment of ascites, but not hepatic encephalopathy

Which of the following discharge instructions would be appropriate for a client who has had a laparoscopic cholecystectomy? 1. Avoid showering for 48 hours after surgery. 2. Return to work within 1 week. 3. Leave dressings in place until you see the surgeon at the postoperative visit. 4. Use acetaminophen (Tylenol) to control any fever

. 3. After a laparoscopic cholecystectomy, the client should not remove dressings from the puncture sites but should wait until visiting the surgeon. The client may shower the day after surgery. A client can return to work within 1 week, but only if approved by the surgeon and no strenuous activity is involved. The client should report any fever, which could be an indication of a complication

A nurse is participating in a diabetes screening program. Who of the following is (are) at risk for developing type 2 diabetes? Select all that apply. 1. A 32-year-old female who delivered a 9½-lb infant. 2. A 44-year-old Native American Indian who has a body mass index (BMI) of 32. 3. An 18-year-old Hispanic who jogs four times a week. 4. A 55-year-old Asian American who has hypertension and two siblings with type 2 diabetes. 5. A 12-year-old who is overweight.

1, 2, 4, 5. The risk factors for developing type 2 diabetes include giving birth to an infant weighing more than 9 lb; obesity (BMI over 30); ethnicity of Asian, African American, or Native American Indian; age greater than 45 years; hypertension; and family history in parents or siblings. Childhood obesity is also a risk factor for type 2 diabetes. Maintaining an ideal weight, eating a low-fat diet, and exercising regularly decrease the risk of type 2 diabetes.

The diabetic educator is teaching a class on diabetes type 1 and is discussing sick-day rules. Which interventions should the diabetes educator include in the discussion? Select all that apply. 1. Take diabetic medication even if unable to eat the client's normal diabetic diet. 2. If unable to eat, drink liquids equal to the client's normal caloric intake. 3. It is not necessary to notify the health-care provider (HCP) if ketones are in the urine. 4. Test blood glucose levels and test urine ketones once a day and keep a record. 5. Call the health-care provider if glucose levels are higher than 180 mg/dL

1, 2, 5. The most important issue to teach clients is to take insulin even if they are unable to eat. Glucose levels are increased with illness and stress. The client should drink liquids such as regular cola or orange juice, or eat regular gelatin, which provide enough glucose to prevent hypoglycemia when receiving insulin. The HCP should be notified if the blood glucose level is this high. Regular insulin may need to be prescribed to keep the blood glucose level within acceptable range.

The nurse is monitoring a client who was diagnosed with type 1 diabetes mellitus and is being treated with NPH and regular insulin. Which manifestations would alert the nurse to the presence of a possible hypoglycemic reaction? Select all that apply. 1. Tremors 2. Anorexia 3. Irritability 4. Nervousness 5. Hot, dry skin 6. Muscle cramps

1, 3, 4. Decreased blood glucose levels produce autonomic nervous system symptoms, which are manifested classically as nervousness, irritability, and tremors. Option 5 is more likely to occur with hyperglycemia. Options 2 and 6 are unrelated to the manifestations of hypoglycemia. In hypoglycemia, usually the client feels hunger

. After a client who has had a laparoscopic cholecystectomy receives discharge instructions, which of the following client statements would indicate that the teaching has been successful? Select all that apply. 1. "I can resume my normal diet when I want." 2. "I need to avoid driving for about 4 weeks." 3. "I may experience some pain in my right shoulder." 4. "I should spend 2 to 3 days in bed before resuming activity." 5. "I can wash the puncture site with mild soap and water."

1, 3, 5. Following a laparoscopic cholecystectomy, the client can resume a normal diet as tolerated. The client may experience right shoulder pain from the gas used to inflate the abdomen. The puncture site should be cleansed daily with mild soap and water. Driving can usually be resumed in 3-4 days follwoing surgery and there is no need to maintain bedrest. Light exercise, such as walking, can be resumed immediately

A client with a diagnosis of addisonian crisis is being admitted to the intensive care unit. Which findings will the interprofessional health care team focus on? Select all that apply. 1. Hypotension 2. Leukocytosis 3. Hyperkalemia 4. Hypercalcemia 5. Hypernatremia

1, 3. In Addison's disease, also known as adrenal insufficiency, destruction of the adrenal gland leads to decreased production of adrenocortical hormones, including the glucocorticoid cortisol and the mineralocorticoid aldosterone. Addisonian crisis, also known as acute adrenal insufficiency, occurs when there is extreme physical or emotional stress and lack of sufficient adrenocortical hormones to manage the stressor. Addisonian crisis is a life-threatening emergency. One of the roles of endogenous cortisol is to enhance vascular tone and vascular response to the catecholamines epinephrine and norepinephrine. Hypotension occurs when vascular tone is decreased and blood vessels cannot respond to epinephrine and norepinephrine. The role of aldosterone in the body is to support the blood pressure by holding salt and water and excreting potassium. When there is insufficient aldosterone, salt and water are lost and potassium builds up; this leads to hypotension from decreased vascular volume, hyponatremia, and hyperkalemia. The remaining options are not associated with addisonian crisis

The nurse is completing an assessment on a client who is being admitted for a diagnostic workup for primary hyperparathyroidism. Which client complaint would be characteristic of this disorder? Select all that apply. 1. Polyuria 2. Headache 3. Bone pain 4. Nervousness 5. Weight gain

1, 3. The role of parathyroid hormone (PTH) in the body is to maintain serum calcium homeostasis. In hyperparathyroidism, PTH levels are high, which causes bone resorption (calcium is pulled from the bones). Hypercalcemia occurs with hyperparathyroidism. Elevated serum calcium levels produce osmotic diuresis and thus polyuria. This diuresis leads to dehydration (weight loss rather than weight gain). Loss of calcium from the bones causes bone pain. Options 2, 4, and 5 are not associated with hyperparathyroidism. Some gastrointestinal symptoms include anorexia, nausea, vomiting, and constipation

When providing care for a client hospitalized with acute pancreatitis who has acute abdominal pain, which of the following nursing interventions would be most appropriate for this client? Select all that apply. 1. Placing the client in a side-lying position. 2. Administering morphine sulfate for pain as needed. 3. Maintaining the client on a high-calorie, high protein diet. 4. Monitoring the client's respiratory status. 5. Obtaining daily weights

1, 4, 5. The client with acute pancreatitis usually experiences acute abdominal pain. Placing the client in a side-lying position relieves the tension on the abdominal area and promotes comfort. A semi-Fowler's position is also appropriate. The nurse should also monitor the client's respiratory status because clients with pancreatitis are prone to develop respiratory complications. Daily weights are obtained to monitor the client's nutritional and fluid volume status. While the client will likely need opioid analgesics to treat the pain, morphine sulfate is not appropriate as it stimulates spasm of the sphincter of Oddi, thus increasing the client's discomfort. During the acute phase of the illness while the client is experiencing pain, the pancreas is rested by withholding food and drink. When the diet is reintroduced, it is a high-carbohydrate, low-fat, bland diet

A client with liver disease presents to the hospital with severe ascites. The nurse caring for the client understands that the pathophysiology involved in the development of ascites includes: Select all that apply. 1. Presence of portal hypertension. 2. Presence of hyperalbuminemia. 3. Increased colloidal osmotic pressure. 4. Sodium and water retention. 5. Presence of hypoaldosteronism.

1, 4. Ascites is the accumulation of plasma-rich fluid in the abdominal cavity. Although portal hypertension is the primary cause of ascites, decreased serum proteins and increased aldosterone also contribute to the fluid accumulation. Hypoalbuminemia (low serum albumin) decreases the colloidal osmotic pressure of plasma. This pressure normally holds fluid in the intravascular compartment, but when the plasma colloidal osmotic pressure decreases, fluid escapes into extravascular compartments. Hyperaldosteronism (an increase in aldosterone)causes sodium and water retention, contributing to ascites and generalized edema.

Which information should the nurse include when developing a teaching plan for a client newly diagnosed with type 2 diabetes mellitus. Select all that apply. 1. A major risk factor for complications is obesity and central abdominal obesity. 2. Supplemental insulin is mandatory for controlling the disease. 3. Exercise increases insulin resistance. 4. The primary nutritional source requiring monitoring in the diet is carbohydrates. 5. Annual eye and foot examinations are recommended by the American Diabetes Association (ADA).

1, 5. Being overweight and having a large waist-hip ratio (central abdominal obesity) increase insulin resistance, making control of diabetes more difficult. The ADA recommends a yearly referral to an ophthalmologist and podiatrist. Exercise and weight management decrease insulin resistance. Insulin is not always needed for type 2 diabetes; diet, exercise, and oral medications are the first-line treatment. The client must monitor all nutritional sources for a balanced diet-fats, carbohydrates, and protein

A nurse is caring for a 35-year-old female client who was recently diagnosed with hypothyroidism. After reviewing the nursing admission assessment, on which documented findings should the nurse plan care for this client? Select all that apply. 1. Hypothermia 2. Hot flashes 3. Nausea 4. Fatigue 5. Tachycardia

1, 5. Symptoms of hypothyroidism include fatigue, increased need for sleep, bradycardia, cold intolerance, ataxia and balance difficulties, and hypothermia. Hot flashes, tachycardia, and nausea are not symptoms of hypothyroidism.

An adult client with type 2 diabetes is taking metformin (Glucophage) 1,000 mg two times every day. After the nurse provides instructions regarding the interaction of alcohol and metformin, the nurse evaluates that the client understands the instructions when the client says: 1. "If I know I'll be having alcohol, I must not take metformin; I could develop lactic acidosis." 2. "If my physician approves, I may drink alcohol with my metformin." 3. "Adverse effects I should watch for are feeling excessively energetic, unusual muscle stiffness, low back pain, and a rapid heartbeat." 4. "If I feel bloated, I should call my physician."

1. A rare but serious adverse effect of metformin (Glucophage) is lactic acidosis; half the cases are fatal. Ideally, one should stop metformin for 2 days before and 2 days after drinking alcohol. Signs and symptoms of lactic acidosis are weakness, fatigue, unusual muscle pain, dyspnea, unusual stomach discomfort, dizziness or light-headedness, and bradycardia or cardiac arrhythmias. Bloating is not an adverse effect of metformin

The home health nurse is completing the admission assessment for a 76-year-old client diagnosed with type 2 diabetes controlled with 70/30 insulin. Which intervention should be included in the plan of care? 1. Assess the client's ability to read small print. 2. Monitor the client's serum prothrombin time (PT) level. 3. Teach the client how to perform a hemoglobin A1c test daily. 4. Instruct the client to check the feet weekly

1. Age-related visual changes and diabetic retinopathy could cause the client to have difficulty in reading and drawing up insulin dosage accurately

The client who has been hospitalized with pancreatitis does not drink alcohol because of her religious convictions. She becomes upset when the physician persists in asking her about alcohol intake. The nurse should explain that the reason for these questions is that: 1. There is a strong link between alcohol use and acute pancreatitis. 2. Alcohol intake can interfere with the tests used to diagnose pancreatitis. 3. Alcoholism is a major health problem, and all clients are questioned about alcohol intake. 4. The physician must obtain the pertinent facts, regardless of religious beliefs.

1. Alcoholism is a major cause of acute pancreatitis in the US. Because some clients are reluctant to discuss alcohol use, staff may inquire about it in several ways. Generally, alcohol intake does not interfere with the tests used to diagnose pancreatitis. Recent ingestion of large amounts of alcohol, however, may cause an increased serum amylase level. Large amounts of ethyl and methyl alcohol may produce an elevated urinary amylase concentration. All clients are asked about alcohol and drug use on hospital admission, but this information is especially pertinent for clients with pancreatitis. Physicians do need to seek facts, but this can be done while respecting the client's religious beliefs. Respecting religious beliefs is important in providing holistic client care

The nurse is working with a morbidly obese client who is seeking help with weight loss at a bariatric clinic. The nurse is planning care and determines that which diagnosis is a priority for this client? 1. Activity Intolerance 2. Disturbed Body Image 3. Defensive Coping 4. Constipation

1. Along with diet, exercise is an important part of a weight loss program. A client with morbid obesity has a sedentary lifestyle and will have activity intolerance. Disturbed Body Image and Constipation may both be legitimate diagnoses, but Activity Intolerance is a greater priority if the client is to lose weight. There is no evidence that this client exhibits defensive coping.

The nurse is teaching the client about home blood glucose monitoring. Which of the following blood glucose measurements indicates hypoglycemia? 1. 59 mg/dL. 2. 75 mg/dL. 3. 108 mg/dL. 4. 119 mg/dL.

1. Although some individual variation exists, when the blood glucose level decreases to less than 70 mg/dL, the client experiences or is at risk for hypoglycemia. Hypoglycemia can occur in both type 1 and type 2 diabetes mellitus, although it is more common when the client is taking insulin. The nurse should instruct the client on the prevention, detection, and treatment of hypoglycemia

Which of the following positions would be appropriate for a client with severe ascites? 1. Fowler's. 2. Side-lying. 3. Reverse Trendelenburg. 4. Sims.

1. Ascites can compromise the action of the diaphragm and increase the client's risk of respiratory problems. Ascites also greatly increases the risk of skin breakdown. Frequent position changes are important, but the preferred position is Fowler's. Placing the client in Fowler's position helps facilitate the client's breathing by relieving pressure on the diaphragm. The other positions do not relieve pressure on the diaphragm

The nurse is concerned that a client with potential hepatic failure is at risk for developing ascites. Which assessment finding would indicate this development? 1. Increased abdominal girth 2. Gallbladder pain 3. Yellow-tinged skin 4. Bleeding and bruising easily

1. Ascites is the accumulation of the fluid in the abdomen, and is a result of liver failure. The client with ascites would have an increased abdominal girth. Jaundice is manifested as yellow-tinged skin, and is the result of hepatic disorders. The client experiencing hepatic problems might have bleeding and bruising issues due to inadequate vitamin K. Obstructed biliary flow could be the cause of gallbladder pain.

The nurse, teaching a class to a group of community members about the importance of weight loss in decreasing the risk of type 2 diabetes mellitus, is asked why weight loss reduces the risk associated with the development of this health problem. Which response by the nurse is most correct? 1. "Excess body weight impairs the body's release of insulin." 2. "The amount of food taken in by those who are overweight requires more insulin to adequately metabolize them, resulting in diabetes." 3. "The physical inactivity associated with obesity causes a reduced ability by the body to produce insulin." 4. "Thin people are less likely to become diabetic."

1. Beta cells of the body release insulin. Their actions are hindered as the amount of adipose tissue in the body increases. The amount of food taken in is not the issue as much as the excess body weight. The body does require more insulin with a greater food intake, but that does not necessarily result in diabetes. While obesity is a risk factor for the development of diabetes, this does not meet the question posed by the client. Inactivity is directly linked to obesity, but it does not present a direct tie to the production of insulin.

The nurse is monitoring a client newly diagnosed with diabetes mellitus for signs of complications. Which sign or symptom, if exhibited in the client, indicates that the client is at risk for chronic complications of diabetes if the blood glucose is not adequately managed? 1. Polyuria 2. Diaphoresis 3. Pedal edema 4. Decreased respiratory rate

1. Chronic hyperglycemia, resulting from poor glycemic control, contributes to the microvascular and macrovascular complications of diabetes mellitus. Classic symptoms of hyperglycemia include polydipsia, polyuria, and polyphagia. Diaphoresis may occur in hypoglycemia. Hypoglycemia is an acute complication of diabetes mellitus; however, it does not predispose a client to the chronic complications of diabetes mellitus. Therefore, option 2 can be eliminated because this finding is characteristic of hypoglycemia. Options 3 and 4 are not associated with diabetes mellitus

A client with cirrhosis is receiving Lactulose (Cephulac). During the assessment the nurse notes increased confusion and asterixis. The nurse should: 1. Assess for GI bleeding. 2. Hold the Lactulose (Cephulac). 3. Increase protein in the diet. 4. Monitor serum bilirubin levels.

1. Clients with cirrhosis can develop hepatic encephalopathy caused by increased ammonia levels. Asterixis, a flapping tremor, is a characteristic symptom of increased ammonia levels. Bacterial action on increased protein in the bowel will increase ammonia levels and cause the encephalopathy to worsen. GI bleeding and protein consumed in the diet increases protein in the intestine and can elevate ammonia levels. Lactulose is given to reduce ammonia formation in the intestine and should not be held since neurological symptoms are worsening. Bilirubin is associated with jaundice.

A client with type 1 diabetes mellitus has influenza. The nurse should instruct the client to: 1. Increase the frequency of self-monitoring (blood glucose testing). 2. Reduce food intake to diminish nausea. 3. Discontinue that dose of insulin if unable to eat. 4. Take half of the normal dose of insulin.

1. Colds and influenza present special challenges to the client with diabetes mellitus because the body's need for insulin increases during illness. Therefore, the client must take the prescribed insulin dose, increase the frequency of blood glucose testing, and maintain an adequate fluid intake to counteract the dehydrating effect of hyperglycemia. Clear fluids, juices, and Gatorade are encouraged. Not taking insulin when sick, or taking half the normal dose, may cause the client to develop ketoacidosis.

Which of the following dietary instructions would be appropriate for the nurse to give a client who is recovering from acute pancreatitis? 1. . Avoid crash dieting. 2. Restrict carbohydrate intake 3. Eat six small meals a day. 4. Decrease sodium in the diet

1. Crash dieting or bingeing may cause an acute attack of pancreatitis and should be avoided. Carbohydrate intake should be increased because carbohydrates are less stimulating to the pancreas. There is no need to maintain a dietary pattern of six meals a day; the client can eat whenever desired. There is no need to place the client on a sodium restricted diet because pancreatitis does not promote fluid retention

A client with type 1 diabetes mellitus is admitted to the emergency department. Which of the following respiratory patterns requires immediate action? 1. Deep, rapid respirations with long expirations. 2. Shallow respirations alternating with long expirations. 3. Regular depth of respirations with frequent pauses. 4. Short expirations and inspirations

1. Deep, rapid respirations with long expirations is indicative of Kussmaul's respirations, which occur in metabolic acidosis. The respirations increase in rate and depth, and the breath has a "fruity" or acetone-like odor. This breathing pattern is the body's attempt to blow off carbon dioxide and acetone, thus compensating for the acidosis. The other breathing patterns listed are not related to ketoacidosis and would not compensate for the acidosis

The nurse is providing care for a 20-year-old client with exophthalmos. Which nursing diagnosis would be the most appropriate for this clinical finding? 1. Disturbed Body Image 2. Ineffective Coping 3. Risk for Injury 4. Activity Intolerance

1. Exophthalmos is a clinical manifestation associated with hyperthyroidism and may be a problem for a young client. The nurse would plan to assess self-esteem and make appropriate referrals. Activity intolerance and risk for injury are not particular to this medical diagnosis. The client's ability to cope could be an issue, but it would probably stem from the disturbed body image.

What diet should be implemented for a client who is in the early stages of cirrhosis? 1. High-calorie, high-carbohydrate. 2. High-protein, low-fat. 3. Low-fat, low-protein. 4. High-carbohydrate, low-sodium

1. For clients who have cirrhosis without complications, a high-calorie, high-carbohydrate diet is preferred to provide an adequate supply of nutrients. In the early stages of cirrhosis, there is no need to restrict fat, protein, or sodium

Which of the following health promotion activities would be appropriate for the nurse to suggest that the client with cirrhosis add to the daily routine at home? 1. Supplement the diet with daily multivitamins. 2. Limit daily alcohol intake. 3. Take a sleeping pill at bedtime. 4. Limit contact with other people whenever possible.

1. General health promotion measures include maintaining good nutrition, avoiding infection, and abstaining from alcohol. Rest and sleep are essential, but an impaired liver may not be able to detoxify sedatives and barbiturates. Such drugs must be used cautiously, if at all, by clients with cirrhosis. The client does not need to limit contact with others but should exercise caution to stay away from ill people.

The nurse administered 28 units of Humulin N, an intermediate-acting insulin, to a client diagnosed with type 1 diabetes at 1600. Which intervention should the nurse implement? 1. Ensure the client eats the bedtime snack. 2. Determine how much food the client ate at lunch. 3. Perform a glucometer reading at 0700. 4. Offer the client protein after administering insulin

1. Humulin N peaks in six (6) to eight (8) hours, making the client at risk for hypoglycemia around midnight, which is why the client should receive a bedtime snack. This snack will prevent nighttime hypoglycemia

The nurse notes that a client with acute pancreatitis occasionally experiences muscle twitching and jerking. How should the nurse interpret the significance of these symptoms? 1. The client may be developing hypocalcemia. 2. The client is experiencing a reaction to meperidine (Demerol). 3. The client has a nutritional imbalance. 4. The client needs a muscle relaxant to help him rest.

1. Hypocalcemia develops in severe cases of acute pancreatitis. The exact cause is unknown. Signs and symptoms of hypocalcemia include jerking and muscle twitching, numbness of fingers and lips, and irritability. Meperidine may cause tremors or seizures as an adverse effect, but not muscle twitching. Muscle twitching is not caused by nutritional deficit, nor does it indicate that the client needs a muscle relaxant

Which of the following is a priority nursing diagnosis for the diabetic client who is taking insulin and has nausea and vomiting from a viral illness or influenza? 1. Imbalanced nutrition: Less than body requirements. 2. Ineffective health maintenance related to ineffective coping skills. 3. Acute pain. 4. Activity intolerance.

1. Imbalanced nutrition: Less than body requirements is a priority nursing diagnosis for the client with diabetes mellitus who is experiencing vomiting with influenza. The diabetic client should eat small, frequent meals of 50 g of carbohydrate or food equal to 200 calories every 3 to 4 hours. If the client cannot eat the carbohydrates or take fluids, the health care provider should be called or the client should go to the emergency department. The diabetic client is in danger of complications with dehydration, electrolyte imbalance, and ketoacidosis. Increasing the client's coping skills is important to lifestyle behaviors, but it is not a priority during this acute illness of influenza. Pain relief may be a need for this client, but it is not the priority at this time; neither is intolerance for activity.

The nurse is preparing a client for a paracentesis. The nurse should: 1. Have the client void immediately before the procedure. 2. Place the client in a side-lying position. 3. Initiate an I.V. line to administer sedatives. 4. Place the client on nothing-by-mouth (NPO) status 6 hours before the procedure

1. Immediately before a paracentesis, the client should empty the bladder to prevent perforation. The client will be placed in a high Fowler's position or seated on the side of the bed for the procedure. I.V. sedatives are not usually administered. The client does not need to be NPO

The nurse is teaching a client with hyperparathyroidism how to manage the condition at home. Which response by the client indicates the need for additional teaching? 1. "I should limit my fluids to 1 liter per day." 2. "I should use my treadmill or go for walks daily." 3. "I should follow a moderate-calcium, high fiber diet." 4. "My alendronate helps to keep calcium from coming out of my bones."

1. In hyperparathyroidism, clients experience excess parathyroid hormone (PTH) secretion. A role of PTH in the body is to maintain serum calcium homeostasis. When PTH levels are high, there is excess bone resorption (calcium is pulled from the bones). In clients with elevated serum calcium levels, there is a risk of nephrolithiasis. One to 2 liters of fluids daily should be encouraged to protect the kidneys and decrease the risk of nephrolithiasis. Moderate physical activity, particularly weight-bearing activity, minimizes bone resorption and helps to protect against pathological fracture. Walking, as an exercise, should be encouraged in the client with hyperparathyroidism. Clients should follow a moderate-calcium, high-fiber diet. Even though serum calcium is already high, clients should follow a moderate-calcium diet because a low-calcium diet will surge PTH. Calcium causes constipation, so a diet high in fiber is recommended. Alendronate is a bisphosphate that inhibits bone resorption. In bone resorption, bone is broken down and calcium is deposited into the serum.

A client with diabetes is taking insulin lispro (Humalog) injections. The nurse should advise the client to eat: 1. Within 10 to 15 minutes after the injection. 2. 1 hour after the injection. 3. At any time, because timing of meals with lispro injections is unnecessary. 4. 2 hours before the injection.

1. Insulin lispro (Humalog) begins to act within 10 to 15 minutes and lasts approximately 4 hours. A major advantage of Humalog is that the client can eat almost immediately after the insulin is administered. The client needs to be instructed regarding the onset, peak, and duration of all insulin, as meals need to be timed with these parameters. Waiting 1 hour to eat may precipitate hypoglycemia. Eating 2 hours before the insulin lispro could cause hyperglycemia if the client does not have circulating insulin to metabolize the carbohydrate.

Which of the following is a priority outcome for the client with Addison's disease? 1. Maintenance of medication compliance. 2. Avoidance of normal activities with stress. 3. Adherence to a 2-g sodium diet. 4. Prevention of hypertensive episodes

1. Medication compliance is an essential part of the self-care required to manage Addison's disease. The client must learn to adjust the glucocorticoid dose in response to the normal and unexpected stresses of daily living. The nurse should instruct the client never to stop taking the drug without consulting the health care provider to avoid an addisonian crisis. Regularity in daily habits makes adjustment easier, but the client should not be encouraged to withdraw from normal activities to avoid stress. The client does not need to restrict sodium. The client is at risk for hyponatremia. Hypotension, not hypertension, is more common with Addison's disease.

The nurse is preparing to administer a.m. medications to clients. Which medication should the nurse question before administering? 1. Pancreatic enzymes to the client who has finished breakfast. 2. The pain medication, morphine, to the client who has a respiratory rate of 20. 3. The loop diuretic to the client who has a serum potassium level of 3.9 mEq/L. 4. The beta blocker to the client who has an apical pulse of 68 bpm

1. Pancreatic enzymes must be administered with meals to enhance the digestion of starches and fats in the gastrointestinal (GI) tract.

A client who has been scheduled to have a choledocholithotomy expresses anxiety about having surgery. Which nursing intervention would be the most appropriate to achieve the outcome of anxiety reduction? 1. Providing the client with information about what to expect postoperatively. 2. Telling the client it is normal to be afraid 3 Reassuring the client by telling her that surgery is a common procedure. 4. Stressing the importance of following the physician's instructions after surgery

1. Providing information can help to answer the client's questions and decrease anxiety. Fear of the unknown can increase anxiety. Telling the client not to be afraid, that the procedure is common, or to follow her physician's orders will not necessarily decrease anxiety

A nurse is working with a female client who is attempting to lose weight. The client admits having difficulty being compliant with the diet prescribed by her doctor. Which suggestions might assist the client in being compliant with the prescribed diet? 1. "Record your food intake so that you can see what and how much you are eating." 2. "Set aside small food reward when you meet a weight loss goal." 3. "Eat alone to reduce outside distractions that may cause you to stray from your diet." 4. "Allow at least 45 minutes to 1 hour to promote full enjoyment of your meal."

1. Recording food intake, amount, location of eating, and situations that induce eating often helps the dieter to gain self-control. Eating is a social activity. Talking with others during mealtime promotes involvement. A meal should be slated to last only 20 minutes. Eating longer can promote eating more. A small nonfood reward can serve as an incentive for working toward a goal. Food rewards should be avoided.

The nurse and an unlicensed assistive personnel (UAP) are caring for clients on an oncology floor. Which intervention should the nurse delegate to the UAP? 1. Assist the client with abdominal pain to turn to the side and flex the knees. 2. Monitor the Jackson Pratt drainage tube to ensure it is draining properly. 3. Check to see if the client is sleeping after pain medication is administered. 4. Empty the bedside commode of the client who has been having melena.

1. The UAP can help a client to turn to the side and assume the fetal position, which decreases some abdominal pain.

The nurse monitors a client with cirrhosis for the development of hepatic encephalopathy. Which of the following would be an indication that hepatic encephalopathy is developing? 1. Decreased mental status. 2. Elevated blood pressure. 3. Decreased urine output. 4. Labored respirations.

1. The client should be monitored closely for changes in mental status. Ammonia has a toxic effect on central nervous system tissue and produces an altered level of consciousness, marked by drowsiness and irritability. If this process is unchecked, the client may lapse into coma. Increasing ammonia levels are not detected by changes in blood pressure, urine output, or respirations.

The client diagnosed with Addison's disease is admitted to the emergency department after a day at the lake. The client is lethargic, forgetful, and weak. Which intervention should the nurse implement? 1. Start an IV with an 18-gauge needle and infuse NS rapidly. 2. Have the client wait in the waiting room until a bed is available. 3. Obtain a permit for the client to receive a blood transfusion. 4. Collect urinalysis and blood samples for a CBC and calcium level

1. The client was exposed to wind and sun at the lake during the hours prior to being admitted to the emergency department. This predisposes the client to dehydration and an addisonian crisis. Rapid IV fluid replacement is necessary.

The client is diagnosed with cancer of the head of the pancreas. Which signs and symptoms should the nurse expect to assess? 1. Clay-colored stools and dark urine. 2. Night sweats and fever. 3. Left lower abdominal cramps and tenesmus. 4. Nausea and coffee-ground emesis

1. The client will have jaundice, clay-colored stools, and tea-colored urine resulting from blockage of the bile drainage

When teaching the diabetic client about foot care, the nurse should instruct the client to do which of the following? 1. Avoid going barefoot. 2. Buy shoes a half size larger. 3. Cut toenails at angles. 4. Use heating pads for sore feet.

1. The client with diabetes is prone to serious foot injuries secondary to peripheral neuropathy and decreased circulation. The client should be taught to avoid going barefoot to prevent injury. Shoes that do not fi t properly should not be worn because they will cause blisters that can become nonhealing, serious wounds for the diabetic client. Toenails should be cut straight across. A heating pad should not be used because of the risk of burns due to insensitivity to temperature.

The nurse is assessing a client with chronic hepatitis B who is receiving Lamivudine (Epivir). What information is most important to communicate to the physician? 1. The client's daily record indicates a 3 kg weight gain over 2 days. 2. The client is complaining of nausea. 3. The client has a temperature of 99° F orally. 4. The client has fatigue.

1. The fluid weight gain is of concern since the drug should be used with caution with impaired renal function. Dosage adjustment may be needed with renal insufficiency since the drug is excreted in the urine. Nausea, mild temperature elevation, and fatigue are symptoms that should be monitored, but are associated with hepatitis.

The nurse should teach the diabetic client that which of the following is the most common symptom of hypoglycemia? 1. Nervousness. 2. Anorexia. 3. Kussmaul's respirations. 4. Bradycardia.

1. The four most commonly reported signs and symptoms of hypoglycemia are nervousness, weakness, perspiration, and confusion. Other signs and symptoms include hunger, incoherent speech, tachycardia, and blurred vision. Anorexia and Kussmaul's respirations are clinical manifestations of hyperglycemia or ketoacidosis. Bradycardia is not associated with hypoglycemia; tachycardia is

The elderly client is admitted to the intensive care department diagnosed with severe HHNS. Which collaborative intervention should the nurse include in the plan of care? 1. Infuse 0.9% normal saline intravenously. 2. Administer intermediate-acting insulin. 3. Perform blood glucometer checks daily. 4. Monitor arterial blood gas (ABG) results.

1. The initial fluid replacement is 0.9% normal saline (an isotonic solution) intravenously, followed by 0.45% saline. The rate depends on the client's fluid volume status and physical health, especially of the heart.

The nurse is planning a home visit for a client with hepatitis. In order to prevent transmission the nurse should focus teaching on: 1. Proper food handling. 2. Insulin syringe disposal. 3. Alpha-interferon. 4. Use of condoms

1. The main route of transmission for hepatitis A is the oral-fecal route, rarely parenteral. Good hand washing before eating or preparing food. Percutaneous transmission is seen with hepatitis B, C, and D. Alpha-interferon is used for treatment of chronic hepatitis B & C

Propylthiouracil (PTU) is prescribed for a client with Graves' disease. The nurse should teach the client to immediately report which of the following? 1. Sore throat. 2. Painful, excessive menstruation. 3. Constipation. 4. Increased urine output.

1. The most serious adverse effects of PTU are leukopenia and agranulocytosis, which usually occur within the first 3 months of treatment. The client should be taught to promptly report to the health care provider signs and symptoms of infection, such as a sore throat and fever. Clients complaining of a sore throat and fever should have an immediate white blood cell count and differential performed, and the drug must be withheld until the results are obtained. Painful menstruation, constipation, and increased urine output are not associated with PTU therapy

The client with Addison's disease is taking glucocorticoids at home. Which of the following statements indicate that the client understands how to take the medication? 1. "Various circumstances increase the need for glucocorticoids, so I will need to adjust the dosage." 2. "My need for glucocorticoids will stabilize and I will be able to take a predetermined dose once a day." 3. "Glucocorticoids are cumulative, so I will take a dose every third day." 4. "I must take a dose every 6 hours to ensure consistent blood levels of glucocorticoids."

1. The need for glucocorticoids changes with circumstances. The basal dose is established when the client is discharged, but this dose covers only normal daily needs and does not provide for additional stressors. As the manager of the medication schedule, the client needs to know signs and symptoms of excessive and insufficient dosages. Glucocorticoid needs fluctuate. Glucocorticoids are not cumulative and must be taken daily. They must never be discontinued suddenly; in the absence of endogenous production, addisonian crisis could result. Two-thirds of the daily dose should be taken at about 8 a.m. and the remainder at about 4 p.m. This schedule approximates the diurnal pattern of normal secretion, with highest levels between 4 a.m. and 6 a.m. and lowest levels in the evening.

When planning care for a client with viral hepatitis, the nurse should review laboratory reports for which of the following abnormal laboratory values? 1. Prolonged prothrombin time. 2. Decreased blood glucose level. 3. Elevated serum potassium level. 4. Decreased serum calcium level

1. The prothrombin time may be prolonged because of decreased absorption of vitamin K and decreased production of prothrombin by the liver. The client should be assessed carefully for bleeding tendencies. Blood glucose and serum potassium and calcium levels are not affected by hepatitis.

The client with diabetes mellitus reports having difficulty cutting his toenails because they are thick and ingrown. What should the nurse recommend to this client? 1. Make an appointment with a podiatrist. 2. Offer to file the tops of the nails to reduce thickness after cutting. 3. Cut the nails straight across with a clipper after the bath. 4. Make an appointment with a nail shop for a pedicure.

1. The toenails of the client with diabetes require close care. If the nails are thick or ingrown, they require the attention of a podiatrist. Cutting the nails across after the bath is correct for toenails that do not demonstrate the complications listed. The client with diabetes is at an increased risk for infection and should avoid situations in which this risk is increased, such as the nail shop pedicure. The nurse should not cut the client's toenails

The nurse is assessing the vital signs of a client experiencing hypoparathyroidism. While monitoring the blood pressure, the nurse notes the client's hand begins to spasm. How should the nurse document this assessment finding? 1. Trousseau sign 2. Chvostek sign 3. Turner's sign 4. Cullen's sign

1. Trousseau sign is elicited when placing a blood pressure cuff on the arm; when the cuff is inflated, the client experiences carpal spasms. Chvostek sign is elicited by tapping on the face in front of the ear and observing for contractions of the muscle. Turner's sign and Cullen's sign are both associated with retroperitoneal bleeding.

The home health nurse visits a client with a diagnosis of type 1 diabetes mellitus. The client relates a history of vomiting and diarrhea and tells the nurse that no food has been consumed for the last 24 hours. Which additional statement by the client indicates a need for further teaching? 1. "I need to stop my insulin." 2. "I need to increase my fluid intake." 3. "I need to monitor my blood glucose every 3 to 4 hours." 4. "I need to call the health care provider (HCP) because of these symptoms."

1. When a client with diabetes mellitus is unable to eat normally because of illness, the client still should take the prescribed insulin or oral medication. The client should consume additional fluids and should notify the HCP. The client should monitor the blood glucose level every 3 to 4 hours. The client should also monitor the urine for ketones during illness.

The nurse is conducting discharge education with a client newly diagnosed with Addison's disease. Which information should be included in the client and family teaching plan? Select all that apply. 1. Addison's disease will resolve over a few weeks, requiring no further treatment. 2. Avoiding stress and maintaining a balanced lifestyle will minimize risk for exacerbations. 3. Fatigue, weakness, dizziness, and mood changes need to be reported to the physician. 4. A medical identification bracelet should be worn. 5. Family members need to be informed about the warning signals of adrenal crisis. 6. Dental work or surgery will require adjustment of daily medication.

2, 3, 4, 5, 6. Addison's disease occurs when the client does not produce enough steroids from the adrenal cortex. Lifetime steroid replacement is needed. The client should be taught lifestyle management techniques to avoid stress and maintain rest periods. A medical identification bracelet should be worn and the family should be taught signs and symptoms that indicate an impending adrenal crisis, such as fatigue, weakness, dizziness, or mood changes. Dental work, infections, and surgery commonly require an adjusted dosage of steroids.

A client with a diagnosis of diabetic ketoacidosis (DKA) is being treated in the emergency department. Which findings support this diagnosis? Select all that apply. 1. Increase in pH 2. Comatose state 3. Deep, rapid breathing 4. Decreased urine output 5. Elevated blood glucose level

2, 3, 5 Rationale: Because of the profound deficiency of insulin associated with DKA, glucose cannot be used for energy and the body breaks down fat as a secondary source of energy. Ketones, which are acid byproducts of fat metabolism, build up and the client experiences a metabolic ketoacidosis. High serum glucose contributes to an osmotic diuresis and the client becomes severely dehydrated. If untreated, the client will become comatose due to severe dehydration, acidosis, and electrolyte imbalance. Kussmaul's respirations, the deep rapid breathing associated with DKA, is a compensatory mechanism by the body. The body attempts to correct the acidotic state by blowing off carbon dioxide (CO2), which is an acid. In the absence of insulin, the client will experience severe hyperglycemia. Option 1 is incorrect because in acidosis the pH would be low. Option 4 is incorrect because a high serum glucose will result in an osmotic diuresis and the client will experience polyuria

A client with diabetes mellitus asks the nurse to recommend something to remove corns from his toes. The nurse should advise the client to: 1. Apply a high-quality corn plaster to the area. 2. Consult a physician or podiatrist about removing the corns. 3. Apply iodine to the corns before peeling them off. 4. Soak the feet in borax solution to peel off the corns.

2. A client with diabetes should be advised to consult a physician or podiatrist for corn removal because of the danger of traumatizing the foot tissue and potential development of ulcers. The diabetic client should never self-treat foot problems but should consult a physician or podiatrist

A 40-year-old client is admitted to the hospital with a diagnosis of acute cholecystitis. The nurse should contact the physician to question which of the following orders? 1. I.V. fluid therapy of normal saline solution to be infused at 100 mL/hour until further orders. 2. Administer morphine sulfate 10 mg I.M. every 4 hours as needed for severe abdominal pain. 3. Nothing by mouth (NPO) until further orders. 4. Insert a nasogastric tube and connect to low intermittent suction.

2. A nurse should question the order for morphine sulfate because it is believed to cause biliary spasm. Thus, the preferred opioid analgesic to treat cholecystitis is meperidine (Demerol). Elderly clients should not be given meperidine because of the risk of acute confusion and seizures in this population. An alternative pain medication will be necessary. I.V. fluid therapy is used to maintain fluid and electrolyte balance that may result from NPO status and gastric suctioning. NPO status and gastric decompression to prevent further gallbladder stimulation

The client with Cushing's disease needs to modify dietary intake to control symptoms. In addition to increasing protein, which strategy would be most appropriate? 1. Increase calories. 2. Restrict sodium. 3. Restrict potassium. 4. Reduce fat to 10%

2. A primary dietary intervention is to restrict sodium, thereby reducing fluid retention. Increased protein catabolism results in loss of muscle mass and necessitates supplemental protein intake. The client may be asked to restrict total calories to reduce weight. The client should be encouraged to eat potassium-rich foods because serum levels are typically depleted. Although reducing fat intake as part of an overall plan to restrict calories is appropriate, fat intake of less than 20% of total calories is not recommended

A client with thyrotoxicosis says to the nurse, "I am so irritable. I am having problems at work because I lose my temper very easily." Which of the following responses by the nurse would give the client the most accurate explanation of her behavior? 1. "Your behavior is caused by temporary confusion brought on by your illness." 2. "Your behavior is caused by the excess thyroid hormone in your system." 3. "Your behavior is caused by your worrying about the seriousness of your illness." 4. "Your behavior is caused by the stress of trying to manage a career and cope with illness.

2. A typical sign of thyrotoxicosis is irritability caused by the high levels of circulating thyroid hormones in the body. This symptom decreases as the client responds to therapy. Thyrotoxicosis does not cause confusion. The client may be worried about her illness, and stress may influence her mood; however, irritability is a common symptom of thyrotoxicosis and the client should be informed of that fact rather than blamed

Which of the following is the priority for a client in addisonian crisis? 1. Controlling hypertension. 2. Preventing irreversible shock. 3. Preventing infection. 4. Relieving anxiety

2. Addison's disease is caused by a deficiency of adrenal corticosteroids and can result in severe hypotension and shock because of uncontrolled loss of sodium in the urine and impaired mineralocorticoid function. This results in loss of extracellular fluid and dangerously low blood volume. Glucocorticoids must be administered to reverse hypotension. Preventing infection is not an appropriate goal of care in this life-threatening situation. Relieving anxiety is appropriate when the client's condition is stabilized, but the calm, competent demeanor of the emergency department staff will be initially reassuring

Which of the following would be an expected finding in a client with adrenal crisis (addisonian crisis)? 1. Fluid retention. 2. Pain. 3. Peripheral edema. 4. Hunger

2. Adrenal hormone deficiency can cause profound physiologic changes. The client may experience severe pain (headache, abdominal pain, back pain, or pain in the extremities). Inhibited gluconeogenesis commonly produces hypoglycemia, and impaired sodium retention causes decreased, not increased, fluid volume. Edema would not be expected. Gastrointestinal disturbances, including nausea and vomiting, are expected findings in Addison's disease, not hunger.

The nurse is preparing a plan of care for a client with diabetes mellitus who has hyperglycemia. The nurse places priority on which client problem? 1. Lack of knowledg 2. Inadequate fluid volume 3. Compromised family coping 4. Inadequate consumption of nutrients

2. An increased blood glucose level will cause the kidneys to excrete the glucose in the urine. This glucose is accompanied by fluids and electrolytes, causing an osmotic diuresis leading to dehydration. This fluid loss must be replaced when it becomes severe. Options 1, 3, and 4 are not related specifically to the information in the question

A 57-year-old with diabetes insipidus is hospitalized for care. Which finding should the nurse report to the physician? 1. Urine output of 350 mL in 8 hours. 2. Urine specific gravity of 1.001. 3. Potassium of 4.0 mEq. 4. Weight gain.

2. Diabetes insipidus is caused by a deficiency of antidiuretic hormone, which results in excretion of a large volume of dilute urine. Therefore, a urine specific gravity of less than 1.005 should be reported. Urine output should be 30 to 50 mL/hour; thus, 350 mL is a normal urinary output over 8 hours. The potassium level is normal. Weight loss, not weight gain, should be monitored as a sign of dehydration.

. The client diagnosed with a pituitary tumor developed syndrome of inappropriate antidiuretic hormone (SIADH). Which interventions should the nurse implement? 1. Assess for dehydration and monitor blood glucose levels. 2. Assess for nausea and vomiting and weigh daily. 3. Monitor potassium levels and encourage fluid intake. 4. Administer vasopressin IV and conduct a fluid deprivation test.

2. Early signs and symptoms are nausea and vomiting. The client has the syndrome of inappropriate secretion of antidiuretic (against allowing the body to urinate) hormone. In other words, the client is producing a hormone that will not allow the client to urinate.

A client with type 1 diabetes mellitus calls the nurse to report recurrent episodes of hypoglycemia with exercising. Which statement by the client indicates an adequate understanding of the peak action of NPH insulin and exercise? 1. "I should not exercise since I am taking insulin." 2. "The best time for me to exercise is after breakfast." 3. "The best time for me to exercise is mid- to late afternoon." 4. "NPH is a basal insulin, so I should exercise in the evening."

2. Exercise is an important part of diabetes management. It promotes weight loss, decreases insulin resistance, and helps to control blood glucose levels. A hypoglycemic reaction may occur in response to increased exercise, so clients should exercise either an hour after mealtime or after consuming a 10- to 15-gram carbohydrate snack, and they should check their blood glucose level before exercising. Option 1 is incorrect because clients with diabetes should exercise, though they should check with their health care provider before starting a new exercise program. Option 3 in incorrect; clients should avoid exercise during the peak time of insulin. NPH insulin peaks at 4 to 12 hours; therefore, afternoon exercise takes place during the peak of the medication. Option 4 is incorrect; NPH insulin in an intermediate-acting insulin, not a basal insulin

After stabilization of Addison's disease, the nurse teaches the client about stress management. The nurse should instruct the client to: 1. Remove all sources of stress from daily life. 2. Use relaxation techniques such as music. 3. Take antianxiety drugs daily. 4. Avoid discussing stressful experiences

2. Finding alternative methods of dealing with stress, such as relaxation techniques, is a cornerstone of stress management. Removing all sources of stress from one's life is not possible. Antianxiety drugs are prescribed for temporary management during periods of major stress, and they are not an intervention in stress management classes. Avoiding discussion of stressful situations will not necessarily reduce stress

The nurse is planning to teach food-drug interactions for a client who was recently diagnosed with hyperthyroidism. Which food should the nurse teach the client to avoid? 1. Caffeine-free soda 2. Soy sauce 3. Milk products 4. High-calorie foods

2. Foods high in iodine, such as soy sauce, can influence the effectiveness of medication therapy for clients who are diagnosed with hyperthyroidism. High-calorie foods are important for clients with hyperthyroidism in order to meet metabolic demands. There is no reason to restrict caffeine-free soda. Milk products should be included in the diet for the client with hyperthyroidism because they are high in protein and calcium.

The nurse is completing a health assessment of a 42-year-old female with suspected Graves' disease. The nurse should assess this client for: 1. Anorexia. 2. Tachycardia. 3. Weight gain. 4. Cold skin

2. Graves' disease, the most common type of thyrotoxicosis, is a state of hypermetabolism. The increased metabolic rate generates heat and produces tachycardia and fi ne muscle tremors. Anorexia is associated with hypothyroidism. Loss of weight, despite a good appetite and adequate caloric intake, is a common feature of hyperthyroidism. Cold skin is associated with hypothyroidism.

. Signs and symptoms of Cushing's disease include: 1. Weight loss. 2. Thin, fragile skin. 3. Hypotension. 4. Abdominal pain.

2. In Cushing's disease, excessive cortisol secretion causes rapid protein catabolism, depleting the collagen support of the skin. The skin becomes thin and fragile and susceptible to easy bruising. The typical "cushingoid" appearance of the client includes a moon face, buffalo hump, central obesity, and thin musculature. Weight gain, mood swings, and slow wound healing are other signs and symptoms of Cushing's disease. Hypertension, not hypotension, is a sign of Cushing's disease. Abdominal pain is not a symptom of Cushing's disease.

Which of the following is normal for a client during the icteric phase of viral hepatitis? 1. Tarry stools. 2. Yellowed sclera. 3. Shortness of breath. 4. Light, frothy urine.

2. Liver inflammation and obstruction block the normal flow of bile. Excess bilirubin turns the skin and sclera yellow and the urine dark and frothy. Profound anorexia is also a common concern. Tarry stools are indicative of gastrointestinal bleeding and would not be expected in hepatitis. Light color stools may occur owing to bile duct obstruction. Shortness of breath is unexpected

The nurse should teach the client with viral hepatitis to: 1. Limit caloric intake and reduce weight. 2. Increase carbohydrates and protein in the diet. 3. Avoid contact with others and live separately. 4. Intensify routine exercise and increase strength.

2. Low-fat, high protein, high carbohydrate diet is encouraged for a client with hepatitis to promote liver rejuvenation. Nutrition intake is important because clients may be anorexic and experience weight loss. Activity should be modified and adequate rest obtained to promote hepatocyte. Social isolation should be avoided and education on preventing transmission should be provided.

he nurse is discharging a client diagnosed with diabetes insipidus. Which statement made by the client warrants further intervention? 1. "I will keep a list of my medications in my wallet and wear a Medic Alert bracelet." 2. "I should take my medication in the morning and leave it refrigerated at home." 3. "I should weigh myself every morning and record any weight gain." 4. "If I develop a tightness in my chest, I will call my health-care provider."

2. Medication for DI is usually taken every eight (8) to 12 hours, depending on the client. The client should keep the medication close at hand

A client is admitted to an emergency department, and a diagnosis of myxedema coma is made. Which action should the nurse prepare to carry out initially? 1. Warm the client. 2. Maintain a patent airway. 3. Administer thyroid hormone. 4. Administer fluid replacement

2. Myxedema coma is a rare but serious disorder that results from persistently low thyroid production. Coma can be precipitated by acute illness, rapid withdrawal of thyroid medication, anesthesia and surgery, hypothermia, and the use of sedatives and opioid analgesics. In myxedema coma, the initial nursing action is to maintain a patent airway. Oxygen should be administered, followed by fluid replacement, keeping the client warm, monitoring vital signs, and administering thyroid hormones by the intravenous route

Bone resorption is a possible complication of Cushing's disease. Which of the following interventions should the nurse recommend to help the client prevent this complication? 1. Increase the amount of potassium in the diet. 2. Maintain a regular program of weight-bearing exercise. 3. Limit dietary vitamin D intake. 4. Perform isometric exercises.

2. Osteoporosis is a serious outcome of prolonged cortisol excess because calcium is resorbed out of the bone. Regular daily weight-bearing exercise (e.g., brisk walking) is an effective way to drive calcium back into the bones. The client should also be instructed to have a dietary or supplemental intake of calcium of 1,500 mg daily. Potassium levels are not relevant to prevention of bone resorption. Vitamin D is needed to aid in the absorption of calcium. Isometric exercises condition muscle tone but do not build bones

A client with diabetes mellitus comes to the clinic for a regular 3-month follow-up appointment. The nurse notes several small bandages covering cuts on the client's hands. The client says, "I'm so clumsy. I'm always cutting my finger cooking or burning myself on the iron." Which of the following responses by the nurse would be most appropriate? 1. "Wash all wounds in isopropyl alcohol." 2. "Keep all cuts clean and covered." 3. "Why don't you have your children do the cooking and ironing?" 4. "You really should be fi ne as long as you take your daily medication.

2. Proper and careful first-aid treatment is important when a client with diabetes has a skin cut or laceration. The skin should be kept supple and as free of organisms as possible. Washing and bandaging the cut will accomplish this. Washing wounds with alcohol is too caustic and drying to the skin. Having the children help is an unrealistic suggestion and does not educate the client about proper care of wounds. Tight control of blood glucose levels through adherence to the medication regimen is vitally important; however, it does not mean that careful attention to cuts can be ignored.

The nurse is caring for a client with cirrhosis of the liver. Which information in the client's health history supports this diagnosis? 1. Smokes two packs of cigarettes per day. 2. Drinks a six-pack of beer each evening. 3. Eats salads for lunch every day. 4. Plays on an adult softball team several times a week.

2. Risk factors for the development of cirrhosis of the liver include excessive alcohol intake. Smoking, ingestion of salads, and exercise are not risk factors for the development of this health problem.

. A client with a large goiter is scheduled for a subtotal thyroidectomy to treat thyrotoxicosis. Saturated solution of potassium iodide (SSKI) is prescribed preoperatively for the client. The primary reason for using this drug is that it helps: 1. Slow progression of exophthalmos. 2. Reduce the vascularity of the thyroid gland. 3. Decrease the body's ability to store thyroxine. 4. Increase the body's ability to excrete thyroxine.

2. SSKI is frequently administered before a thyroidectomy because it helps decrease the vascularity of the thyroid gland. A highly vascular thyroid gland is very friable, a condition that presents a hazard during surgery. Preparation of the client for surgery includes depleting the gland of thyroid hormone and decreasing vascularity. SSKI does not decrease the progression of exophthalmos, and it does not decrease the body's ability to store thyroxine or increase the body's ability to excrete thyroxine

The nurse should monitor the client with Cushing's disease for which of the following? 1. Postprandial hypoglycemia. 2. Hypokalemia. 3. Hyponatremia. 4. Decreased urine calcium level.

2. Sodium retention is typically accompanied by potassium depletion. Hypertension, hypokalemia, edema, and heart failure may result from the hypersecretion of aldosterone. The client with Cushing's disease exhibits postprandial or persistent hyperglycemia. Clients with Cushing's disease have hypernatremia, not hyponatremia. Bone resorption of calcium increases the urine calcium level

A client with cirrhosis begins to develop ascites. Spironolactone (Aldactone) is prescribed to treat the ascites. The nurse should monitor the client closely for which of the following drug-related adverse effects? 1. Constipation. 2. Hyperkalemia. 3. Irregular pulse. 4. Dysuria.

2. Spironolactone (Aldactone) is a potassium sparing diuretic; therefore, clients should be monitored closely for hyperkalemia. Other common adverse effects include abdominal cramping, diarrhea, dizziness, headache, and rash. Constipation and dysuria are not common adverse effects of spironolactone. An irregular pulse is not an adverse effect of spironolactone but could develop if serum potassium levels are not closely monitored

The nurse measures the amount of bile drainage from a T-tube and records it by which one of the following methods? 1. Adding it to the client's urine output. 2. Charting it separately on the output record. 3. Adding it to the amount of wound drainage. 4. Subtracting it from the total intake for each day

2. T-tube bile drainage is recorded separately on the output record. Adding the T-tube drainage to the urine output or wound drainage makes it difficult to accurately determine the amounts of bile, urine, or drainage. The client's total intake will be incorrect if drainage is subtracted from it.

Which of the following symptoms might indicate that a client was developing tetany after a subtotal thyroidectomy? 1. Pains in the joints of the hands and feet. 2. Tingling in the fingers. 3. Bleeding on the back of the dressing. 4. Tension on the suture line.

2. Tetany may occur after thyroidectomy if the parathyroid glands are accidentally injured or removed during surgery. This would cause a disturbance in serum calcium levels. An early sign of tetany is numbness and tingling of the fingers or toes and in the circumoral region. Tetany may occur from 1-7 days postoperatively. Late signs and symptoms of tetany include seizures, contraction of the glottis, and respiratory obstruction. Pains in the joints of the hands and feet are not early symptoms of tetany. Bleeding on the back of the dressing is related to possible incisional complications. Tension on the suture line may indicate swelling, infection, or internal bleeding, but it is not related to tetany

Which of the following expected outcomes would be appropriate for a client with viral hepatitis? The client will: 1. Demonstrate a decrease in fluid retention related to ascites. 2. Verbalize the importance of reporting bleeding gums or bloody stools. 3. Limit use of alcohol to two to three drinks per week. 4. Restrict activity to within the home to prevent disease transmission

2. The client should be able to verbalize the importance of reporting any bleeding tendencies that could be the result of a prolonged prothrombin time. Ascites is not typically a clinical manifestation of hepatitis; it is associated with cirrhosis. Alcohol use should be eliminated for at least 1 year after the diagnosis of hepatitis to allow the liver time to fully recover. There is no need for a client to be restricted to the home because hepatitis is not spread through casual contact between individuals

Which of the following indicates a potential complication of diabetes mellitus? 1. Inflamed, painful joints. 2. Blood pressure of 160/100 mm Hg. 3. Stooped appearance. 4. Hemoglobin of 9 g/dL.

2. The client with diabetes mellitus is especially prone to hypertension due to atherosclerotic changes, which leads to problems of the microvascular and macrovascular systems. This can result in complications in the heart, brain, and kidneys. Heart disease and stroke are twice as common among people with diabetes mellitus than among people without the disease. Painful, inflamed joints accompany rheumatoid arthritis. A stooped appearance accompanies osteoporosis with narrowing of the vertebral column. A low hemoglobin concentration accompanies anemia, especially iron deficiency anemia and anemia of chronic disease.

Which of the following medications should be available to provide emergency treatment if a client develops tetany after a subtotal thyroidectomy? 1. Sodium phosphate. 2. Calcium gluconate. 3. Echothiophate iodide. 4. Sodium bicarbonate.

2. The client with tetany is suffering from hypocalcemia, which is treated by administering an I.V. preparation of calcium, such as calcium gluconate or calcium chloride. Oral calcium is then necessary until normal parathyroid function returns. Sodium phosphate is a laxative. Echothiophate iodide is an eye preparation used as a miotic for an antiglaucoma effect. Sodium bicarbonate is a potent systemic antacid.

Which assessment data indicate the client diagnosed with diabetic ketoacidosis is responding to the medical treatment? 1. The client has tented skin turgor and dry mucous membranes. 2. The client is alert and oriented to date, time, and place. 3. The client's ABG results are pH 7.29, Paco2 44, HCO3 15. 4. The client's serum potassium level is 3.3 mEq/L.

2. The client's level of consciousness can be altered because of dehydration and acidosis. If the client's sensorium is intact, the client is getting better and responding to the medical treatment.

The nurse is preparing an education session for nurses who work in an endocrinology clinic that cares for the older clients. Which characteristic of hypothyroidism should the nurse include for this group? 1. Thyroid hormone is often increased in the elderly. 2. Symptoms of hypothyroidism in the elderly are often confused with symptoms of aging. 3. Hypothyroidism in the elderly is a congenital disease. 4. The elderly client with hypothyroidism presents with pitting edema.

2. The nurse educator must emphasize that the risk with elderly clients is that the diagnosis of hypothyroidism can be missed as symptoms of aging may mask those of the disease. Not all hypothyroidism is congenital, and it is inaccurate to state that older clients develop the disease due to congenital defects. Thyroid hormone is decreased in all clients with hypothyroidism. The adult client will present with non-pitting edema.

An 18-year-old female client, 5′4″ tall, weighing 113 kg, comes to the clinic for a nonhealing wound on her lower leg, which she has had for two (2) weeks. Which disease process should the nurse suspect the client has developed? 1. Type 1 diabetes. 2. Type 2 diabetes. 3. Gestational diabetes. 4. Acanthosis nigricans.

2. Type 2 diabetes is a disorder usually occurring around the age of 40, but it is now being detected in children and young adults as a result of obesity and sedentary lifestyles. Nonhealing wounds are a hallmark sign of type 2 diabetes. This client weighs 248.6 pounds and is short.

60-year-old female is diagnosed with hypothyroidism. The nurse should assess the client for which of the following? 1. Tachycardia. 2. Weight gain. 3. Diarrhea. 4. Nausea.

2. Typical signs and symptoms of hypothyroidism include weight gain, fatigue, decreased energy, apathy, brittle nails, dry skin, cold intolerance, hair loss, constipation, and numbness and tingling in the fi ngers. Tachycardia is a sign of hyperthyroidism, not hypothyroidism. Diarrhea and nausea are not symptoms of hypothyroidism.

The nurse is assessing the feet of a client with long-term type 2 diabetes. Which assessment data warrant immediate intervention by the nurse? 1. The client has crumbling toenails. 2. The client has athlete's foot. 3. The client has a necrotic big toe. 4. The client has thickened toenails

3. A necrotic big toe indicates "dead" tissue. The client does not feel pain, does not realize the injury, and does not seek treatment. Increased blood glucose levels decrease the oxygen supply needed to heal the wound and increase the risk for developing an infection.

he nurse should assess a client with Addison's disease for which of the following? 1. Weight gain. 2. Hunger. 3. Lethargy. 4. Muscle spasms

3. Although many of the disease signs and symptoms are vague and nonspecific, most clients experience lethargy and depression as early symptoms. Other early signs and symptoms include mood changes, emotional lability, irritability, weight loss, muscle weakness, fatigue, nausea, and vomiting. Most clients experience a loss of appetite. Muscles become weak, not spastic, because of adrenocortical insufficiency.

A client is admitted with acute necrotizing pancreatitis. Lab results have been obtained and a peripheral I.V. has been inserted. Which of the following orders from a health care provider should the nurse question? 1. Infuse a 500 mL normal saline bolus. 2. Calcium gluconate 90 mg in 100 mL NS. 3. Total parenteral nutrition (TPN) at 72 mL/ hour. 4. Placement of a Foley catheter

3. Clients with acute necrotizing pancreatitis should remain NPO with early enteral feedings via the jejunum to maintain bowel integrity and immune function. TPN is considered if enteral feedings are contraindicated. Fat necrosis occurring with acute pancreatitis can cause hypocalcemia requiring calcium replacement. A Foley catheter provides accurate output assessment to monitor for prerenal acute renal failure that can occur from hypovolemia.

The nurse is providing discharge instructions for a client with cirrhosis. Which of the following statements best indicates that the client has understood the teaching? 1. "I should eat a high-protein, high-carbohydrate diet to provide energy." 2. "It is safer for me to take acetaminophen for pain instead of aspirin." 3. "I should avoid constipation to decrease chances of bleeding." 4. "If I get enough rest and follow my diet, it is possible for my cirrhosis to be cured.

3. Clients with cirrhosis should be instructed to avoid constipation and straining at stool to prevent hemorrhage. The client with cirrhosis has bleeding tendencies because of the liver's inability to produce clotting factors. A low-protein and high-carbohydrate diet is recommended. Clients with cirrhosis should not take acetaminophen (Tylenol), which is potentially hepatotoxic. Aspirin also should be avoided if esophageal varices are present. Cirrhosis is a chronic disease

The client diagnosed with type 2 diabetes is admitted to the intensive care unit (ICU) with hyperosmolar hyperglycemic nonketotic syndrome (HHNS) coma. Which assessment data should the nurse expect the client to exhibit? 1. Kussmaul's respirations. 2. Diarrhea and epigastric pain. 3. Dry mucous membranes. 4. Ketone breath odor.

3. Dry mucous membranes are a result of the hyperglycemia and occur with both HHNS and DKA

A client with type 1 diabetes mellitus has diabetic ketoacidosis. Which of the following findings has the greatest effect on fluid loss? 1. Hypotension. 2. Decreased serum potassium level. 3. Rapid, deep respirations. 4. Warm, dry skin

3. Due to the rapid, deep respirations, the client is losing fluid from vaporization from the lungs and skin (insensible fluid loss). Normally, about 900 mL of fluid is lost per day through vaporization. Decreased serum potassium level has no effect on insensible fluid loss. Hypotension occurs due to polyurea and inadequate fluid intake. It may decrease the flow of blood to the skin, causing skin to be warm and dry

The nurse is assessing a client who is in the early stages of cirrhosis of the liver. Which focused assessment is appropriate? 1. Peripheral edema. 2. Ascites. 3. Anorexia. 4. Jaundice.

3. Early clinical manifestations of cirrhosis are subtle and usually include gastrointestinal symptoms, such as anorexia, nausea, vomiting, and changes in bowel patterns. These changes are caused by the liver's altered ability to metabolize carbohydrates, proteins, and fats. Peripheral edema, ascites, and jaundice are later signs of liver failure and portal hypertension

A client with Addison's disease is admitted to the medical unit. The nurse diagnoses the client with Deficient fluid volume related to inadequate fluid intake and to fluid loss secondary to inadequate adrenal hormone secretion. As the client's oral intake increases, which of the following fluids would be most appropriate? 1. Milk and diet soda. 2. Water and eggnog. 3. Bouillon and juice. 4. Coffee and milkshakes

3. Electrolyte imbalances associated with Addison's disease include hypoglycemia, hyponatremia, and hyperkalemia. Salted bouillon and fruit juices provide glucose and sodium to replenish these deficits. Diet soda does not contain sugar. Water could cause further sodium dilution. Coffee's diuretic effect would aggravate the fluid defi cit. Milk contains potassium and sodium

The nurse is teaching a diabetic client using an empowerment approach. The nurse should initiate teaching by asking which of the following? 1. "How much does your family need to be involved in learning about your condition?" 2. "What is required for your family to manage your symptoms?" 3. "What activities are most important for you to be able to maintain control of your diabetes?" 4. "What do you know about your medications and condition?

3. Empowerment is an approach to clinical practice that emphasizes helping people discover and use their innate abilities to gain mastery over their own condition. Empowerment means that individuals with a health problem have the tools, such as knowledge, control, resources, and experience, to implement and evaluate their self-management practices. Involvement of others, such as asking the client about family involvement, implies that the others will provide the direct care needed rather than the client. Asking the client what the client needs to know implies that the nurse will be the one to provide the information. Telling the client what is required does not provide the client with options or lead to empowerment

. Cushing's disease is manifested by the excessive secretion of corticosteroids. The hormones involved are: 1. Glucocorticoids and aldosterone. 2. Adrenocorticotropic hormone (ACTH). 3. Glucocorticoids, aldosterone, and androgens. 4. Catecholamines.

3. Excessive levels of glucocorticoids, aldosterone, and androgens secreted from the adrenal cortex result in the constellation of symptoms known as Cushing's disease. Cushing's disease can be caused by a tumor, overstimulation from the pituitary, or the use of prescription steroid drugs. Androgens are also secreted in excess. ACTH is only one hormone that is abnormal in Cushing's disease. Excessive secretion of catecholamines accompanies pheochromocytoma, a disease of the adrenal medulla.

An older female client with hyperthyroidism is not a candidate for surgery. Which treatment could be used to quickly reduce the client's manifestations of the disorder? 1. Nothing, because there is little effect on the quality of life in older adults. 2. A partial thyroidectomy 3. The ingestion of radioactive sodium iodine, I131 4. A combination treatment with levothyroid (Synthroid) and amiodarone (Cordarone)

3. Hyperthyroidism in older clients is most often caused by Graves disease ortoxic nodular goiter, which is diagnosed by subnormal or undetectable levels of TSH (thyroid-stimulating hormone). The treatment of choice is the ingestion of radioactive iodine, which is picked up by the thyroid tissue and then destroys the tissue. This treatment avoids surgery, anesthesia, and hospitalization. Surgical removal is reserved for clients with symptoms too severe for treatment with radioactive iodine or nodules suspicious for malignancy. Treatment with levothyroid (Synthroid) would worsen symptoms, and amiodarone (Cordarone) is an antiarrhythmic drug that has induced hyperthyroidism in some clients. Older clients with hyperthyroidism seldom present with the classic symptoms seen in younger clients, but older clients may develop cardiac dysrhythmias, weight loss, fatigue, and apathy, and do require treatment. A partial thyroidectomy is a surgical procedure, for which the client is not a candidate

A nurse is teaching a client with type 1 diabetes mellitus who jogs daily about the preferred sites for insulin absorption. What is the most appropriate site for a client who jogs? 1. Arms. 2. Legs. 3. Abdomen. 4. Iliac crest

3. If the client engages in an activity or exercise that focuses on one area of the body, that area may cause inconsistent absorption of insulin. A good regimen for a jogger is to inject the abdomen for 1 week and then rotate to the buttock. A jogger may have inconsistent absorption in the legs or arms with strenuous running. The iliac crest is not an appropriate site due to a lack of loose skin and subcutaneous tissue in that area.

A client undergoes a laparoscopic cholecystectomy. Which of the following dietary instructions should the nurse give the client immediately after surgery? 1. "You cannot eat or drink anything for 24 hours." 2. "You may resume your normal diet the day after your surgery." 3. "Drink liquids today and eat lightly for a few days." 4. "You can progress from a liquid to a bland diet as tolerated."

3. Immediately after surgery, the client will drink liquids. A light diet can be resumed the day after surgery. There is no need for the client to remain on nothing-by-mouth status after surgery because peristaltic bowel activity should not be affected. The client will probably not be able to tolerate a full meal comfortably the day after surgery. There is no need for the client to stay on a bland diet after a laparoscopic cholecystectomy. The client should, however, avoid excessive fats.

A client's serum ammonia level is elevated, and the physician orders 30 mL of lactulose (Cephulac). Which of the following is an adverse effect of this drug? 1. Increased urine output. 2. Improved level of consciousness. 3. Increased bowel movements. 4. Nausea and vomiting.

3. Lactulose increases intestinal motility, thereby trapping and expelling ammonia in the feces. An increase in the number of bowel movements is expected as an adverse effect. Lactulose does not affect urine output. Any improvements in mental status would be the result of increased ammonia elimination, not an adverse effect of the drug. Nausea and vomiting are not common adverse effects of lactulose

The nurse asks the client to state her name as soon as she regains consciousness postoperatively after a subtotal thyroidectomy and at each assessment. The nurse does this to monitor for signs of which of the following? 1. Internal hemorrhage. 2. Decreasing level of consciousness. 3. Laryngeal nerve damage. 4. Upper airway obstruction.

3. Laryngeal nerve damage is a potential complication of thyroid surgery because of the proximity of the thyroid gland to the recurrent laryngeal nerve. Asking the client to speak helps assess for signs of laryngeal nerve damage. Persistent or worsening hoarseness and weak voice are signs of laryngeal nerve damage and should be reported to the physician immediately. Internal hemorrhage is detected by changes in vital signs. The client's level of consciousness can be partially assessed by asking her to speak, but that is not the primary reason for doing so in this situation. Upper airway obstruction is detected by color and respiratory rate and pattern.

The nurse is planning a program for clients at a health fair regarding the prevention and early detection of cancer of the pancreas. Which self-care activity should the nurse discuss as an example of a primary nursing intervention? 1. Monitor for elevated blood glucose at random intervals. 2. Inspect the skin and sclera of the eyes for a yellow tint. 3. Limit meat in the diet and eat a diet low in fat. 4. Instruct the client with hyperglycemia about insulin injections

3. Limiting the intake of meat and fats in the diet is an example of primary interventions. Risk factors for the development of cancer of the pancreas are cigarette smoking and eating a high-fat diet. By changing these behaviors, the client could possibly prevent the development of cancer of the pancreas. Other risk factors include genetic predisposition and exposure to industrial chemicals

66. The nurse is caring for clients on a medical floor. Which client should be assessed first? 1. The client diagnosed with syndrome of inappropriate antidiuretic hormone (SIADH) who has a weight gain of 1.5 pounds since yesterday. 2. The client diagnosed with a pituitary tumor who has developed diabetes insipidus (DI) and has an intake of 1,500 mL and an output of 1,600 mL in the last 8 hours. 3. The client diagnosed with syndrome of inappropriate antidiuretic hormone (SIADH) who is having muscle twitching. 4. The client diagnosed with diabetes insipidus (DI) who is complaining of feeling tired after having to get up at night

3. Muscle twitching is a sign of early sodium imbalance. If an immediate intervention is not made, the client could begin to seize

When teaching a client newly diagnosed with primary Addison's disease, the nurse should explain that the disease results from: 1. Insufficient secretion of growth hormone (GH). 2. Dysfunction of the hypothalamic pituitary. 3. Idiopathic atrophy of the adrenal gland. 4. Oversecretion of the adrenal medulla

3. Primary Addison's disease refers to a problem in the gland itself that results from idiopathic atrophy of the glands. The process is believed to be autoimmune in nature. The most common causes of primary adrenocortical insufficiency are autoimmune destruction (70%) and tuberculosis (20%). Insufficient secretion of GH causes dwarfism or growth delay. Hyposecretion of glucocorticoids, aldosterone, and androgens occur with Addison's disease. Pituitary dysfunction can cause Addison's disease, but this is not a primary disease process. Oversecretion of the adrenal medulla causes pheochromocytoma.

Which of the following signs and symptoms would probably indicate that the client with Addison's disease is receiving too much glucocorticoid replacement? 1. Anorexia. 2. Dizziness. 3. Rapid weight gain. 4. Poor skin turgor.

3. Rapid weight gain, because it reflects excess fluids, is a warning sign that the client is receiving too much hormone replacement. It may be difficult to individualize the correct dosage for a client taking glucocorticoids, and the therapeutic range between underdosage and overdosage is narrow. Maintaining the client on the lowest dose that provides satisfactory clinical response is always the goal of pharmacotherapeutics. Fluid balance is an important indicator of the adequacy of hormone replacement. Anorexia is not present with glucocorticoid therapy because these drugs increase the appetite. Dizziness is not specific to the effects of glucocorticoid therapy. Poor skin turgor is a late sign of fluid volume deficit

The client received 10 units of Humulin R, a fast-acting insulin, at 0700. At 1030 the unlicensed assistive personnel (UAP) tells the nurse the client has a headache and is really acting "funny." Which intervention should the nurse implement first? 1. Instruct the UAP to obtain the blood glucose level. 2. Have the client drink eight (8) ounces of orange juice. 3. Go to the client's room and assess the client for hypoglycemia. 4. Prepare to administer one (1) ampule 50% dextrose intravenously

3. Regular insulin peaks in two (2) to four (4) hours. Therefore, the nurse should think about the possibility the client is having a hypoglycemic reaction and should assess the client. The nurse should not delegate nursing tasks to a UAP if the client is unstable

he nurse is administering a saturated solution of potassium iodide (SSKI). The nurse should: 1. Pour the solution over ice chips. 2. Mix the solution with an antacid. 3. Dilute the solution with water, milk, or fruit juice and have the client drink it with a straw. 4. Disguise the solution in a pureed fruit or vegetable

3. SSKI should be diluted well in milk, water, juice, or a carbonated beverage before administration to help disguise the strong, bitter taste. Also, this drug is irritating to mucosa if taken undiluted. The client should sip the diluted preparation through a drinking straw to help prevent staining of the teeth. Pouring the solution over ice chips will not sufficiently dilute the SSKI or cover the taste. Antacids are not used to dilute or cover the taste of SSKI. Mixing in a puree would put the SSKI in contact with the teeth.

A client reports that she has gained weight and that her face and body are "rounder," while her legs and arms have become thinner. A tentative diagnosis of Cushing's disease is made. When examining this client, the nurse would expect to find: 1. Orthostatic hypotension. 2. Muscle hypertrophy in the extremities. 3. Bruised areas on the skin. 4. Decreased body hair.

3. Skin bruising from increased skin and blood vessel fragility is a classic sign of Cushing's disease. Hyperpigmentation and bruising are caused by the hypersecretion of glucocorticoids. Fluid retention causes hypertension, not hypotension. Muscle wasting occurs in the extremities. Hair on the head thins, while body hair increases

A 58-year-old client who is newly diagnosed with type 2 diabetes has smoked for 30 years. When teaching the client on ways to optimize health outcomes, what should the nurse explain about the effects of smoking and diabetes? 1. Smoking is a major factor in the development of diabetic neuropathy. 2. Smoking increases insulin resistance. 3. Smoking accelerates arteriosclerotic changes in blood vessels. 4. Smoking promotes weight gain.

3. Smoking is especially unhealthy for diabetic clients because smoking accelerates the arteriosclerotic effects that occur in blood vessels from elevated levels of blood glucose. Smoking is not associated with weight gain; in fact, people use weight gain as an excuse not to quit smoking. Poor glycemic control in diabetics is associated with the development of complications including diabetic neuropathy. Smoking does not affect insulin resistance.

Which of the following conditions is the most significant risk factor for the development of type 2 diabetes mellitus? 1. Cigarette smoking. 2. High-cholesterol diet. 3. Obesity. 4. Hypertension

3. The most important factor predisposing to the development of type 2 diabetes mellitus is obesity. Insulin resistance increases with obesity. Cigarette smoking is not a predisposing factor, but it is a risk factor that increases complications of diabetes mellitus. A high-cholesterol diet does not necessarily predispose to diabetes mellitus, but it may contribute to obesity and hyperlipidemia. Hypertension is not a predisposing factor, but it is a risk factor for developing complications of diabetes mellitus.

A client is brought to the emergency department in an unresponsive state, and a diagnosis of hyperosmolar hyperglycemic syndrome is made. The nurse would immediately prepare to initiate which anticipated health care provider's prescription? 1. Endotracheal intubation 2. 100 units of NPH insulin 3. Intravenous infusion of normal saline 4. Intravenous infusion of sodium bicarbonate

3. The primary goal of treatment in hyperosmolar hyperglycemic syndrome (HHS) is to rehydrate the client to restore fluid volume and to correct electrolyte deficiency. Intravenous (IV) fluid replacement is similar to that administered in diabetic ketoacidosis (DKA) and begins with IV infusion of normal saline. Regular insulin, not NPH insulin, would be administered. The use of sodium bicarbonate to correct acidosis is avoided because it can precipitate a further drop in serum potassium levels. Intubation and mechanical ventilation are not required to treat HHS

A client is admitted to the hospital with a diagnosis of cholecystitis from cholelithiasis. The client has severe abdominal pain, nausea, and has vomited several times. Based on these data, which nursing diagnosis would have the highest priority for intervention at this time? 1. Anxiety related to severe abdominal discomfort. 2. Deficient fluid volume related to vomiting. 3. Pain related to gallbladder inflammation. 4. Imbalanced nutrition: Less than body requirements related to vomiting

3. The priority for nursing care at this time is to decrease the client's severe abdominal pain, which is frequently accompanied by nausea and vomiting, is caused by biliary spasm. Opioid analgesics are given to relieve the severe pain and spasm of cholecystitis. Relief of pain may decrease nausea and vomiting and thereby decrease the client's likelihood of developing further complications, such as deficient fluid volume and imbalanced nutrition. There is no data to suggest the client is anxious

The nurse is planning a teaching seminar for a group of young adult client who are at risk for obesity. What should the nurse include in the program for this group? 1. There are drugs that are good to use to reduce weight. 2. Obesity puts the client at risk for anorexia nervosa. 3. Proper diet and exercise programs 4. The obese client will eventually be bulimic.

3. The young adults who are at risk for obesity need education about changing lifestyles and the importance of preventing obesity as opposed to treating it. Education should include tips on eating healthy and exercising. Drugs used for obesity are not without risk. Bulimia and anorexia nervosa are not eating disorders that are caused by obesity.

. The charge nurse of an intensive care unit is making assignments for the night shift. Which client should be assigned to the most experienced intensive care nurse? 1. The client diagnosed with respiratory failure who is on a ventilator and requires frequent sedation. 2. The client diagnosed with lung cancer and iatrogenic Cushing's disease with ABGs of pH 7.35, Pao2 88, Paco2 44, and HCO3 22. 3. The client diagnosed with Addison's disease who is lethargic and has a BP of 80/45, P 124, and R 28. 4. The client diagnosed with hyperthyroidism who has undergone a thyroidectomy two (2) days ago and has a negative Trousseau's sign.

3. This client has a low blood pressure and tachycardia. This client may be experiencing an addisonian crisis, a potentially life-threatening condition. The most experienced nurse should care for this client.

A client has just been admitted to the nursing unit following thyroidectomy. Which assessment is the priority for this client? 1. Hypoglycemia 2. Level of hoarseness 3. Respiratory distress 4. Edema at the surgical sit

3. Thyroidectomy is the removal of the thyroid gland, which is located in the anterior neck. It is very important to monitor airway status, as any swelling to the surgical site could cause respiratory distress. Although all of the options are important for the nurse to monitor, the priority nursing action is to monitor the airway

The nurse should teach the client to prevent corneal irritation from mild exophthalmos by: 1. Massaging the eyes at regular intervals. 2. Instilling an ophthalmic anesthetic as ordered. 3. Wearing dark-colored glasses. 4. Covering both eyes with moistened gauze pads

3. Treatment of mild ophthalmopathy that may accompany thyrotoxicosis includes measures such as wearing sunglasses to protect the eyes from corneal irritation. Treatment of ophthalmopathy should be performed in consultation with an ophthalmologist. Massaging the eyes will not help to protect the cornea. An ophthalmic anesthetic is used to examine and possibly treat a painful eye, not protect the cornea. Covering the eyes with moist gauze pads is not a satisfactory nursing measure to protect the eyes of a client with exophthalmos because treatment is not focused on moisture to the eye but rather on protecting the cornea and optic nerve. In exophthalmos, the retrobulbar connective tissues and extraocular muscle volume are expanded because of fluid retention. The pressure is also increased

The nurse is assessing a client with hepatitis and notices that the AST and ALT lab values have increased. Which of the following statements by the client requires further instruction by the nurse? 1. "I require increased periods of rest." 2. "I follow a low-fat, high carbohydrate diet." 3. "I eat dry toast to relieve my nausea." 4. "I take acetaminophen (Tylenol) for arthritis pain."

4. Acetaminophen is toxic to the liver and should be avoided in a client with liver dysfunction. Increased periods of rest allow for liver regeneration. A low-fat, high carbohydrate diet and dry toast to relieve nausea are appropriate.

The nurse is instructing a young adult with Addison's disease how to adjust the dose of glucocorticoids. The nurse should explain that the client may need an increased dosage of glucocorticoids in which of the following situations? 1. Completing the spring semester of school. 2. Gaining 4 pounds. 3. Becoming engaged. 4. Undergoing a root canal

4. Adrenal crisis can occur with physical stress, such as surgery, dental work, infection, flu, trauma, and pregnancy. In these situations, glucocorticoid and mineralocorticoid dosages are increased. Weight loss, not gain, occurs with adrenal insufficiency. Psychological stress has less effect on corticosteroid need than physical stress.

The nurse is discussing the importance of exercising with a client diagnosed with type 2 diabetes whose diabetes is well controlled with diet and exercise. Which information should the nurse include in the teaching about diabetes? 1. Eat a simple carbohydrate snack before exercising. 2. Carry peanut butter crackers when exercising. 3. Encourage the client to walk 20 minutes three (3) times a week. 4. Perform warm-up and cool-down exercises.

4. All clients who exercise should perform warm-up and cool-down exercises to help prevent muscle strain and injury.

An external insulin pump is prescribed for a client with diabetes mellitus. When the client asks the nurse about the functioning of the pump, the nurse bases the response on which information about the pump? 1. It is timed to release programmed doses of either short-duration or NPH insulin into the bloodstream at specific intervals. 2. It continuously infuses small amounts of NPH insulin into the bloodstream while regularly monitoring blood glucose levels. 3. It is surgically attached to the pancreas and infuses regular insulin into the pancreas. This releases insulin into the bloodstream. 4. It administers a small continuous dose of short duration insulin subcutaneously. The client can self-administer an additional bolus dose from the pump before each meal

4. An insulin pump provides a small continuous dose of short-duration (rapid- or short-acting) insulin subcutaneously throughout the day and night. The client can self-administer an additional bolus dose from the pump before each meal as needed. Short-duration insulin is used in an insulin pump. An external pump is not attached surgically to the pancreas

The nurse should teach the client with Addison's disease that the adverse effect of bronze colored skin is thought to be caused by which of the following? 1. Hypersensitivity to sun exposure. 2. Increased serum bilirubin level. 3. Adverse effects of the glucocorticoid therapy. 4. Increased secretion of adrenocorticotropic hormone (ACTH).

4. Bronzing, or general deepening of skin pigmentation, is a classic sign of Addison's disease and is caused by melanocyte-stimulating hormone produced in response to increased ACTH secretion. The hyperpigmentation is typically found in the distal portion of extremities and in areas exposed to sun. Additionally, areas that may not be exposed to sun, such as the nipples, genitalia, tongue, and knuckles, become bronze-colored. Treatment of Addison's disease usually reverses the hyperpigmentation. Bilirubin level is not related to the pathophysiology of Addison's disease. Hyperpigmentation is not related to the effects of the glucocorticoid therapy.

The nurse is discussing a new diet order with a client who is seeking help to lose weight. The client asks the nurse how to best balance the new diet. What should the nurse respond to this client? 1. "Your diet should consist of 1,250-1,500 calories per day, with 15% of the calories being sources of protein." 2. "Your diet should be consist of 750-1,000 calories per day, with less than 15% of the total calories coming from fat." 3. "Your diet should simply cut 500 calories per day from your normal intake." 4. "Your diet should consist of 1,000-1,200 calories per day, with less than 15% of the total calories coming from fat."

4. Collaboration with a nutritionist helps clients to identify healthy foods that appeal to them and that can make up a diet plan to create a daily 500- to 1,000-kcal deficit. Ideally, the recommended diet should be low in kilocalories and fat, contain adequate nutrients and minerals, and be high in dietary fiber. The client should eat regular meals with small servings. A gradual, slow weight loss of no more than 1-2 lb/week is recommended. For most individuals, this means a diet of 1,000-1,200 kcal/day for most women and 1,200-1,600 kcal/day for men. Fewer than 1,200 kcal each day may lead to loss of lean tissue and nutritional deficiencies

The nurse is planning a staff development program on how to care for clients with hepatitis A. Which of the following precautions should the nurse indicate as essential when caring for clients with hepatitis A? 1. Gowning when entering a client's room. 2. Wearing a mask when providing care. 3. Assigning the client to a private room. 4. Wearing gloves when giving direct care

4. Contact precautions are recommended for clients with hepatitis A. This includes wearing gloves for direct care. These recommendations are made by the Centers for Disease Control and Prevention. A gown is not required unless substantial contact with the client is anticipated. It is not necessary to wear a mask. The client does not need a private room unless incontinent of stool

The client with diabetes mellitus says, "If I could just avoid what you call carbohydrates in my diet, I guess I would be okay." The nurse should base the response to this comment on the knowledge that diabetes affects metabolism of which of the following? 1. Carbohydrates only. 2. Fats and carbohydrates only. 3. Protein and carbohydrates only. 4. Proteins, fats, and carbohydrates

4. Diabetes mellitus is a multifactorial, systemic disease associated with problems in the metabolism of all food types. The client's diet should contain appropriate amounts of all three nutrients, plus adequate minerals and vitamins

Assessment of the diabetic client for common complications should include examination of the: 1. Abdomen. 2. Lymph glands. 3. Pharynx. 4. Eyes

4. Diabetic retinopathy, cataracts, and glaucoma are common complications in diabetics, necessitating eye assessment and examination. The feet should also be examined at each client encounter, monitoring for thickening, fissures, or breaks in the skin; ulcers; and thickened nails. Although assessments of the abdomen, pharynx, and lymph glands are included in a thorough examination, they are not pertinent to common diabetic complications.

The nurse provides instructions to a client newly diagnosed with type 1 diabetes mellitus. The nurse recognizes accurate understanding of measures to prevent diabetic ketoacidosis when the client makes which statement? 1. "I will stop taking my insulin if I'm too sick to eat." 2. "I will decrease my insulin dose during times of illness." 3. "I will adjust my insulin dose according to the level of glucose in my urine." 4. "I will notify my health care provider (HCP) if my blood glucose level is higher than 250 mg/dL (14.2 mmol/L)."

4. During illness, the client with type 1 diabetes mellitus is at increased risk of diabetic ketoacidosis, due to hyperglycemia associated with the stress response and due to a typically decreased caloric intake. As part of sick day management, the client with diabetes should monitor blood glucose levels and should notify the HCP if the level is higher than 250 mg/dL(14.2 mmol/ L). Insulin should never be stopped. In fact, insulin may need to be increased during times of illness. Doses should not be adjusted without the HCP's advice and are usually adjusted on the basis of blood glucose levels, not urinary glucose readings

Serum concentrations of thyroid hormones and thyroid-stimulating hormone (TSH) are tests ordered for the client with thyrotoxicosis. Which of the following laboratory values are indicative of thyrotoxicosis? 1. Elevated thyroid hormone concentrations and normal TSH. 2. Elevated TSH and normal thyroid hormone concentrations. 3. Decreased thyroid hormone concentrations and elevated TSH. 4. Elevated thyroid hormone concentrations and decreased TSH

4. Elevated serum concentrations of thyroid hormones and suppressed serum TSH are the features of thyrotoxicosis. Decreased or absent serum TSH is a very accurate indicator of thyrotoxicosis. Increased levels of circulating thyroid hormones cause the feedback mechanism to the brain to suppress TSH secretion.

A client who has undergone a subtotal thyroidectomy is subject to complications in the first 48 hours after surgery. The nurse should obtain and keep at the bedside equipment to: 1. Begin total parenteral nutrition. 2. Start a cutdown infusion. 3. Administer tube feedings. 4. Perform a tracheotomy.

4. Equipment for an emergency tracheotomy should be kept in the room, in case tracheal edema and airway occlusion occur. Laryngeal nerve damage can result in vocal cord spasm and respiratory obstruction. A tracheostomy set, oxygen and suction equipment, and a suture removal set (for respiratory distress from hemorrhage) make up the emergency equipment that should be readily available. Total parenteral nutrition is not anticipated for the client undergoing thyroidectomy. Intravenous infusion via a cutdown is not an expected possible treatment after thyroidectomy. Tube feedings are not anticipated emergency care

The nurse is completing an assessment interview with a client being seen for a yearly physical examination. Which client statement would indicate a possible diagnosis of diabetes? 1. "I'm slightly winded when I walk up a flight of stairs, but it passes quickly." 2. "I feel a bit tired by mid-afternoon and take a 30-minute nap most days." 3. "I sometimes have muscle aches in my upper legs at night." 4. "I've been experiencing increased thirst during the past several months."

4. Excessive thirst can be associated with high glucose levels and may be a symptom of undiagnosed diabetes mellitus. Fatigue that responds to a short nap, having some muscle aches at night, and being slightly short of breath after walking up a flight of stairs with a quick recovery may be within the normal functioning of a healthy older client

College freshman are participating in a study abroad program. When teaching them about hepatitis B, the nurse should instruct the students on: 1. Water sanitation. 2. Single dormitory rooms. 3. Vaccination for hepatitis D. 4. Safe sexual practices.

4. Hepatitis B is considered a sexually transmitted disease and students should observe safe sex practices. Poor sanitary conditions in underdeveloped countries relate to spread of hepatitis A and E. Focusing on routes of transmission and avoidance of infection can prevent the spread of hepatitis; isolation in single rooms is not required. There is no vaccine for hepatitis D.

The nurse is preparing a community education program about preventing hepatitis B infection. Which of the following would be appropriate to incorporate into the teaching plan? 1. Hepatitis B is relatively uncommon among college students. 2. Frequent ingestion of alcohol can predispose an individual to development of hepatitis B. 3. Good personal hygiene habits are most effective at preventing the spread of hepatitis B. 4. The use of a condom is advised for sexual intercourse

4. Hepatitis B is spread through exposure to blood or blood products and through high-risk sexual activity. Hepatitis B is considered to be a sexually transmitted disease. High-risk sexual activities include sex with multiple partners, unprotected sex with an infected individual, male homosexual activity, and sexual activity with I.V. drug users. The Centers for Disease Control and Prevention recommends immunization of all newborns and adolescents. College students are at high risk for development of hepatitis B and are encouraged to be immunized. Alcohol intake by itself does not predispose an individual to hepatitis B, but it can lead to high-risk behaviors such as unprotected sex. Good personal hygiene alone will not prevent the transmission of hepatitis B

When discussing recent onset of feelings of sadness and depression in a client with hypothyroidism, the nurse should inform the client that these feelings are: 1. The effects of thyroid hormone replacement therapy and will diminish over time. 2. Related to thyroid hormone replacement therapy and will not diminish over time. 3. A normal part of having a chronic illness. 4. Most likely related to low thyroid hormone levels and will improve with treatment

4. Hypothyroidism may contribute to sadness and depression. It is good practice for clients with newly diagnosed depression to be monitored for hypothyroidism by checking serum thyroid hormone and thyroid-stimulating hormone levels. This client needs to know that these feelings may be related to her low thyroid hormone levels and may improve with treatment. Replacement therapy does not cause depression. Depression may accompany chronic illness, but it is not "normal."

The nurse is caring for a client admitted to the emergency department with diabetic ketoacidosis (DKA). In the acute phase, the nurse plans for which priority intervention? 1. Correct the acidosis. 2. Administer 5% dextrose intravenously. 3. Apply a monitor for an electrocardiogram. 4. Administer short-duration insulin intravenously

4. Lack of insulin is the primary cause of DKA. Treatment consists of insulin administration (short- or rapid-acting), intravenous fluid administration (normal saline initially, not 5% dextrose), and potassium replacement, followed by correcting acidosis. Cardiac monitoring is important due to alterations in potassium levels associated with DKA and its treatment, but applying an electrocardiogram monitor is not the priority action.

After a cholecystectomy, the client is to follow a low-fat diet. Which of the following foods would be most appropriate to include in a low-fat diet? 1. Cheese omelet. 2. Peanut butter. 3. Ham salad sandwich 4. Roast beef

4. Lean meats, such as beef, lamb, veal, and well-trimmed lean ham and pork, are low in fat. Rice, pasta, and veggies are low in fat when not served with butter, cream, or sauces. Fruits are low in fat. The amount of fat allowed in a postop cholecystectomy diet depends on tolerance. Typically, the client does not require a special diet but is encouraged to avoid excessive dat intake. A cheese omelet and peanut butter have high fat. Ham salad sandwich is high in fat due to the dressing

Which of the following is the best indicator for determining whether a client with Addison's disease is receiving the correct amount of glucocorticoid replacement? 1. Skin turgor. 2. Temperature. 3. Thirst. 4. Daily weight.

4. Measuring daily weight is a reliable, objective way to monitor fluid balance. Rapid variations in weight reflect changes in fluid volume, which suggests insufficient control of the disease and the need for more glucocorticoids in the client with Addison's disease. Nurses should instruct clients taking oral steroids to weigh themselves daily and to report any unusual weight loss or gain. Skin turgor testing does supply information about fluid status, but daily weight monitoring is more reliable. Temperature is not a direct measurement of fluid balance. Thirst is a nonspecific and very late sign of weight loss.

A client has advanced cirrhosis of the liver. The client's spouse asks the nurse why his abdomen is swollen, making it very difficult for him to fasten his pants. How should the nurse respond to provide the most accurate explanation of the disease process? 1. "He must have been eating too many foods with salt in them. Salt pulls water with it." 2. "The swelling in his ankles must have moved up closer to his heart so the fluid circulates better." 3. "He must have forgotten to take his daily water pill." 4. "Blood is not able to flow readily through the liver now, and the liver cannot make protein to keep fluid inside the blood vessels."

4. Portal hypertension and hypoalbuminemia as a result of cirrhosis cause a fluid shift into the peritoneal space causing ascites. In a cardiac or kidney problem, not cirrhosis, sodium can promote edema formation and subsequent decreased urine output. Edema does not migrate upward toward the heart to enhance its circulation. Although diuretics promote the excretion of excess fluid, occasionally forgetting or omitting a dose will not yield the ascites found in cirrhosis of the liver

A client with Graves' disease is treated with radioactive iodine (RAI) in the form of sodium iodide 131I. Which of the following statements by the nurse will explain to the client how the drug works? 1. "The radioactive iodine stabilizes the thyroid hormone levels before a thyroidectomy." 2. "The radioactive iodine reduces uptake of thyroxine and thereby improves your condition." 3. "The radioactive iodine lowers the levels of thyroid hormones by slowing your body's production of them." 4. "The radioactive iodine destroys thyroid tissue so that thyroid hormones are no longer produced."

4. Sodium iodide 131I destroys the thyroid follicular cells, and thyroid hormones are no longer produced. RAI is commonly recommended for clients with Graves' disease, especially the elderly. The treatment results in a "medical thyroidectomy." RAI is given in lieu of surgery, not before surgery. RAI does not reduce uptake of thyroxine. The outcome of giving RAI is the destruction of the thyroid follicular cells. It is possible to slow the production of thyroid hormones with RAI.

The nurse suspects that a client is experiencing hypothyroidism. Which question should the nurse ask to assess for this disorder? 1. "Is your skin often clammy?" 2. "Do you have brown, shiny patches on your legs?" 3. "Is your skin smooth?" 4. "Is your skin rough and dry?"

4. The client experiencing hypothyroidism has rough, dry skin. Smooth skin is associated with hyperthyroidism. Cool, clammy skin is found in clients with low blood sugar. Brown, shiny patches on the lower extremities are associated with poor circulation.

he nurse is caring for a client diagnosed with diabetes insipidus (DI). Which intervention should be implemented? 1. Administer sliding-scale insulin as ordered. 2. Restrict caffeinated beverages. 3. Check urine ketones if blood glucose is >250. 4. Assess tissue turgor every four (4) hours.

4. The client is excreting large amounts of dilute urine. If the client is unable to drink enough fluids, the client will quickly become dehydrated, so tissue turgor should be assessed frequently

The nurse should monitor the client with acute pancreatitis for which of the following complications? 1. Heart failure. 2. Duodenal ulcer 3. Cirrhosis. 4. Pneumonia

4. The client with acute pancreatitis is prone to complications associated with the respiratory system. Pneumonia, atelectasis, and pleural effusion are examples of respiratory complications that can develop as a result of pancreatic enzyme exudate. Pancreatitis does not cause heart failure, ulcer formation, or cirrhosis.

The client with type 1 diabetes mellitus is taught to take isophane insulin suspension NPH (Humulin N) at 5 p.m. each day. The client should be instructed that the greatest risk of hypoglycemia will occur at about what time? 1. 11 a.m., shortly before lunch. 2. 1 p.m., shortly after lunch. 3. 6 p.m., shortly after dinner. 4. 1 a.m., while sleeping.

4. The client with diabetes mellitus who is taking NPH insulin (Humulin N) in the evening is most likely to become hypoglycemic shortly after midnight because this insulin peaks in 6 to 8 hours. The client should eat a bedtime snack to help prevent hypoglycemia while sleeping.

Which nursing intervention should be included in the plan of care for the client diagnosed with hyperthyroidism? 1. Increase the amount of fiber in the diet. 2. Encourage a low-calorie, low-protein diet. 3. Decrease the client's fluid intake to 1,000 mL/day. 4. Provide six (6) small, well-balanced meals a day

4. The client with hyperthyroidism has an increased appetite; therefore, well-balanced meals served several times throughout the day will help with the client's constant hunger.

The nurse assesses that the client with hepatitis is experiencing fatigue, weakness, and a general feeling of malaise. The client tires rapidly during morning care. Based on this information, which of the following would be an appropriate nursing diagnosis? 1. Impaired physical mobility related to malaise. 2. Self-care deficit related to fatigue. 3. Ineffective coping related to long-term illness 4. Activity intolerance related to fatigue

4. The most appropriate diagnosis for this client is Activity intolerance related to fatigue. The major goal of care for the client with hepatitis is to increase activity gradually as tolerated. Periods of alternating rest and activity should be included in the plan of care. There is no evidence that the client is physically immobile, unable to provide self-care, or coping ineffectively.

The nurse is preparing to teach a client who is newly diagnosed with type 1 diabetes mellitus on the preferred area to self-inject insulin. On which area should the nurse focus based upon insulin absorption rates? 1. Deltoid 2. Thigh 3. Hip 4. Abdomen

4. The rate of absorption and peak of action of insulin differ according to the site. The site that allows the most rapid absorption is the abdomen, followed by the deltoid muscle, then the thigh, and then the hip. Because of the rapid absorption, the abdomen is the recommended site.

The client diagnosed with HHNS was admitted yesterday with a blood glucose level of 780 mg/ dL. The client's blood glucose level is now 300 mg/dL. Which intervention should the nurse implement? 1. Increase the regular insulin IV drip. 2. Check the client's urine for ketones. 3. Provide the client with a therapeutic diabetic meal. 4. Notify the HCP to obtain an order to decrease insulin

4. When the glucose level is decreased to around 300 mg/dL, the regular insulin infusion therapy is decreased. Subcutaneous insulin will be administered per sliding scale.

A client newly diagnosed with type 1 diabetes mellitus tells the nurse that the diagnosis must be wrong because the client is not overweight, eats all of the time, and is thin. What should the nurse respond to the client? 1. "Thin people can be diabetic, too." 2. "Your condition makes it impossible for you to gain weight." 3. "Your lab tests indicate the presence of diabetes." 4. "You are eating large quantities because your condition makes it difficult for your body to obtain energy from the foods taken in."

4. he diabetic client is unable to obtain the needed glucose for the body's cells, due to the lack of insulin. Patients diagnosed with type 1 diabetes mellitus experience polyphagia, and are often thin. While the statement about diabetics being thin is correct, it does not answer the client. It is not impossible for diabetics to gain weight. Although the laboratory tests might indicate the presence of diabetes, this does not meet the client's needs for teaching.

A client has an open cholecystectomy with bile duct exploration. Following surgery, the client has a T-tube. To evaluate the effectiveness of the T-tube, the nurse should: 1. Irrigate the tube with 20 mL of normal saline every 4 hours. 2. Unclamp the T-tube and empty the contents every day. 3. Assess the color and amount of drainage every shift. 4. Monitor the multiple incision sites for bile drainage

A T-tube is inserted in the common bile duct to maintain patency until edema from the duct exploration subsides. The bile color should be gold to dark green and the amount should be closely monitored to ensure tube patency. Irrigation is not routinely done, unless ordered. The T-tube is not clamped in the early postop period to allow for continuous drainage, An open cholecystectomy has one right subcostal incision, where a laparoscopic cholecystectomy has multiple small icnisions.

A client's stools are light gray in color. The nurse should assess the client further for which of the following? Select all that apply. 1. Intolerance to fatty foods. 2. Fever. 3. Jaundice. 4. Respiratory distress. 5. Pain at McBurney's point. 6. Peptic ulcer disease.

Bile is created in the liver, stored in the gallbladder, and released into the duodenum giving stool its brown color. A bile duct obstruction can cause pale colored stools. Other symptoms associated with cholelithiasis are right upper quadrant tenderness, fever from inflammation or infection, jaundice from elevated serum bilirubin levels, and nausea or right upper quadrant pain after a fatty meal. Pain at McBurney's point lies between the umbilics and right iliac crest and is associated with appendicitis. A bleeding ulcer produced black, tarry stools. Respiratory distress is not a symptom of cholelithiasis.

The client is diagnosed with acute pancreatitis. Which health-care provider's admitting order should the nurse question? 1. Bedrest with bathroom privileges. 2. Initiate IV therapy of D5W at 125 mL/hr. 3. Weigh the client daily. 4. Low-fat, low-carbohydrate diet.

The client will be NPO, which will decrease stimulation of the pancreatic enzymes, resulting in decreased autodigestion of the pancreas, therefore decreasing pain.


Kaugnay na mga set ng pag-aaral

Consumers in a Global Market: Chapter 7 Global Fashion Retailing and Tourism

View Set

BUSL 2000 Chapter 24: Employment and Discrimination Law

View Set

Real Estate Pre-Licensing Section 6

View Set

Exam 3 (chapter 15, part of chapter 16)

View Set

Ap European History: Chapter 16 Scientific Revolution Vocab

View Set

Chapter 18- Personality Disorders

View Set

Ch. 12 - Substance-Related and Addictive Disorders

View Set

Bus Mgmt Ch 10 Managing the Forms of Business Ownership

View Set